SlideShare a Scribd company logo
1 of 150
ANORECTAL DISEASES
SATHIYAMURTHY LOGESHWARRAJ
17-0691-378
BLOCK 10
GROUP 5
CLERKSHIP INTERNATIONAL MARCH BATCH 2021
SURGERY ROTATION
DEPARTMENT OF SURGERY
JONELTA FOUNDATION SCHOOL OF MEDICINE
UNIVERSITY OF PEPETUAL HELP RIZAL
1. Discuss the basic anatomy of the anal
canal and rectum
Anatomy of the Anal Canal and Rectum
Landmarks
● The rectum is approximately 12 to 15 cm in length
● Valves of Houston
○ Three distinct submucosal folds that extend into the rectal
lumen.
● Posteriorly, the presecral fascia separates the rectum from the
presacral venous plexus and the pelvic nerves.
Anatomy of the Anal Canal and Rectum
● At S4, the rectosacral fascia (Waldeyer’s fascia) extends anteriorly
and caudally and attaches to the fascia propria at the anorectal
junction.
● Anteriorly, Denonvilliers’ fascia separates the rectum from the
prostate and seminal vesicles in men and from the vagina in women.
● The lateral ligaments support the lower rectum
Anatomy of the Anal Canal and Rectum
The Anatomic Anal Canal
● Extends from the dentate or pectinate line to the anal verge
● The dentate or pectinate line marks the transition point between
columnar rectal mucosa and squamous anoderm.
● The anal transition zone
○ includes mucosa proximal to the dentate line that shares histologic
characteristics of columnar, cuboidal, and squamous epithelium.
○ Extends only 1 to 2 cm proximal to the dentate line, however this
value is highly variable and can be as far as 15 cm proximal to the
dentate line.
Anatomy of the Anal Canal and Rectum
The Anatomic Anal Canal
● The dentate line is surrounded by longitudinal mucosal folds, known
as the columns of Morgagni, into which the anal crypts empty
○ These crypts are the source of cryptoglandular abscesses
Anatomy of the Anal Canal and Rectum
The Surgical Anal Canal
● Begins at the anorectal junction and terminates at the anal verge.
● Measures 2 to 4 cm in length and is generally longer in men than in
women.
Anatomy of the Anal Canal and Rectum
Distal Rectum
● The inner smooth muscle is thickened and comprises the internal
anal sphincter that is surrounded by the subcutaneous, superficial,
and deep external sphincter.
● The deep external anal sphincter is an extension of the puborectalis
muscle.
● The puborectalis, iliococcygeus, and pubococcygeus muscles form
the levator ani muscle of the pelvic floor
Anorectal Arterial Supply
● The superior rectal artery
○ arises from the terminal
branch of the inferior
mesenteric artery
○ supplies the upper rectum.
● The middle rectal artery
○ arises from the internal
iliac
○ the presence and size of
these arteries are highly
variable.
Anorectal Arterial Supply
● The inferior rectal artery
○ arises from the internal
pudendal artery (a branch
of the internal iliac artery)
● A rich network of collaterals
connects the terminal
arterioles of each of these
arteries, thus making the
rectum relatively resistant to
ischemia
Anorectal Venous Drainage
● The venous drainage of the
rectum parallels the arterial
supply.
● The superior rectal vein drains
into the portal system via the
inferior mesenteric vein.
● The middle rectal vein drains
into the internal iliac vein.
Anorectal Venous Drainage
● The inferior rectal vein drains
into the internal pudendal
vein, and subsequently into the
internal iliac vein.
● A submucosal plexus deep to
the columns of Morgagni forms
the hemorrhoidal plexus and
drains into all three veins.
Lymphatic Drainage
Rectal
● Parallels the vascular supply.
● Lymphatic channels in the upper and middle rectum drain superiorly
into the inferior mesenteric lymph nodes.
● Lymphatic channels in the lower rectum drain both superiorly into
the inferior mesenteric lymph nodes and laterally into the internal
iliac lymph nodes.
Lymphatic Drainage
Anal Canal
● More complex pattern of lymphatic drainage.
● Proximal to the dentate line, lymph drains into both the inferior
mesenteric lymph nodes and the internal iliac lymph nodes.
● Distal to the dentate line, lymph primarily drains into the inguinal
lymph nodes, but can also drain into the inferior mesenteric lymph
nodes and internal iliac lymph nodes.
Anorectal Nerve Supply
● Both sympathetic and parasympathetic nerves innervate the
anorectum.
○ Sympathetic nerve fibers are derived from L1–L3 and join the
preaortic plexus.
■ The preaortic nerve fibers then extend below the aorta to
form the hypogastric plexus, which subsequently joins the
parasympathetic fibers to form the pelvic plexus.
Anorectal Nerve Supply
○ Parasympathetic nerve fibers are known as the nervi erigentes
and originate from S2–S4
■ These fibers join the sympathetic fibers to form the pelvic
plexus.
● Sympathetic and parasympathetic fibers then supply the anorectum
and adjacent urogenital organs.
Anorectal Nerve Supply
● The internal anal sphincter
○ innervated by sympathetic and parasympathetic nerve fibers
■ both types of fibers inhibit sphincter contraction
● The external anal sphincter and puborectalis muscles
○ innervated by the inferior rectal branch of the internal pudendal
nerve.
● The levator ani
○ Innervated by both the internal pudendal nerve and direct
branches of S3 to S5.
Anorectal Nerve Supply
● Sensory innervation to the anal canal
○ provided by the inferior rectal branch of the pudendal nerve.
○ While the rectum is relatively insensate, the anal canal below
the dentate line receives somatic innervation
Anorectal Nerve Supply
2. Discuss the basics of the
mechanism of defecation
Motility
● Unlike the small intestine, the large intestine does not demonstrate cyclic
motor activity characteristic of the migratory motor complex.
● The colon displays intermittent contractions of either low or high
amplitude.
● Low-amplitude, short-duration contractions occur in bursts and appear to
move the colonic contents both antegrade and retrograde.
● It is thought that these bursts of motor activity delay colonic transit and
thus increase the time available for absorption of water and exchange of
electrolytes.
Motility
● High-amplitude, prolonged duration, propagated contractions (HAPCs)
● Occur in a more coordinated fashion and create “mass movements,” four to
ten times per day, mostly after meals and awakening.
● Bursts of “rectal motor complexes” also have been described.
● In general, cholinergic activation increases colonic motility.
Defecation
● Defecation is a complex, coordinated mechanism involving colonic mass movement,
increased intra-abdominal and rectal pressure, and relaxation of the pelvic floor.
● Distention of the rectum causes a reflex relaxation of the internal anal sphincter (the
rectoanal inhibitory reflex) that allows the contents to make contact with the anal
canal.
● This “sampling reflex” allows the sensory epithelium to distinguish solid stool from
liquid stool and gas.
● If defecation does not occur, the rectum relaxes and the urge to defecate passes
(accommodation response).
● Defecation proceeds by coordinating increasing intra-abdominal pressure via a
Valsalva maneuver with rectal contraction, relaxation of the puborectalis muscle, and
opening of the anal canal.
Continence
● Continence requires:
○ Adequate rectal wall compliance to accommodate the fecal bolus
○ Appropriate neurogenic control of the pelvic floor and sphincter mechanism
○ Functional internal and external sphincter muscles
● At rest, the puborectalis muscle creates a “sling” around the distal rectum,
forming a relatively acute angle that distributes intra-abdominal forces
onto the pelvic floor.
○ With defecation, this angle straightens, allowing downward force to be applied along
the axis of the rectum and anal canal.
● The internal and external sphincters are tonically active at rest.
○ The internal sphincter is responsible for most of the resting, involuntary sphincter
tone (resting pressure).
○ The external sphincter is responsible for most of the voluntary sphincter tone
(squeeze pressure).
Continence
● Branches of the pudendal nerve innervate both the internal and external
sphincter.
● The hemorrhoidal cushions may contribute to continence by mechanically
blocking the anal canal.
● Finally, liquid stools exacerbate abnormalities with these anatomic and
physiologic mechanisms, so a formed stool contributes to maintaining
continence.
○ Thus, impaired continence may result from:
■ poor rectal compliance
■ injury to the internal
■ and/or external sphincter or puborectalis, or neuropathy
3. Describe and perform proper
evaluation of anorectal diseases.
Complete History and Physical Examination
· Starting points for evaluation of any patient complaining with symptoms related to diseases
of the colon, rectum and anus
· Past medical history
· Surgical History to detect underlying conditions
o Resultant gastrointestinal anatomy
o Anorectal surgery – abdominal or anorectal complaints
· Obstetric history – occult pelvic floor and/or anal sphincter damage
· Family history – colorectal disease, especially inflammatory bowel disease, polyps, and
colorectal cancer
o History of other malignancies – genetic syndrome
· Medications – some may cause gastrointestinal symptoms
o Adequacy of medical treatment
· Visual inspection of anus and perineum
o Digital rectal examination
Endoscopy
Anoscopy
· Instrument used to examine the anal canal
· Measures approximately 8 cm in length
· Larger anoscope – anal procedures
o Rubber band ligation
o Inserted into the anal canal
o Obturator is withdrawn, inspection is done and the anoscope withdrawn
o Rotated 90 degrees and reinserted to allow visualization of all four quadrants of
the canal
o If patient can’t tolerate DRE, anoscopy should not be attempted
Proctoscopy
· Rigid proctoscope – examination of rectum and distal sigmoid colon
· Ocassionally used to therapeutics
· 25 cm in length and in various diameters (15 or 19mm)
· Pediatric (11 mm)
· Suction is necessary for an adequate examination
· Transanal Endoscopic Microsurgery (TEM)
o Much wider diameter
o Can be used for excision of large polyps and tumors
· Transanal Minimally Invasice Surgery (TAMIS)
o Can achieve similar resections to TEM but does not utilize a proctoscope and
depends on insufflation
Flexible Sigmoidoscopy and Colonoscopy
· Video or fiberoptic sigmoidoscopy and colonoscopy
· Sigmoidoscopes measures 60cm in length
· Full depth may allow visualization until the splenic flexure
· Partial preparation with enemas is adequate
· Most patients can tolerate this without sedation
· Colonoscopes measure 100-160cm in length can examine the the entire colon and terminal
ileum
· Complete bowel preparation is usually necessary
· Duration and discomfort usually requires conscious sedation
· Electrocautery should not be used in the absence of complete bowel preparation
o Risk for explosion of intestinal methane or hydrogen gases
· Colonoscopes –single channel where snares, biopsy forceps or electrocautery can be passed
o Suction and irrigation capability
o Therapeutic colonoscopies possess two channels – simultaneous suction/irrigation and
use of snares, biopsy forceps and electrocautery
Capsule Endoscopy
· Small ingestible camera
· Images of mucosa and GIT are captured
· Transmitted by radiofrequency to a belt-held receiver and to the computer
· Used to detect small bowel lesions
· Possibility of an acute obstruction led to:
o Dissolvable capsule
Imaging
Plain X-rays and Contrast Studies
· Plain x-rays (supine, upright and diaphragmatic views)
o Detects free intra-abdominal air
o Bowel gas patterns – obstruction
o Volvulus
· Obstructive symptoms
· Delineating fistulous tracts
· Diagnosing small perforations
· Anastomotic leaks
· Gastrografin cannot provide mucosal detail provided by barium, this water-soluble agent is
recommended for perforation or leak
· Double-contrast barium enema (followed by insufflation of air)
o 70-90% sensitive mass lesions greater than 1cm in diameter
Computed Tomography
· Detection of extraluminal disease
o Intra-abdominal abscesses
o Pericolic inflammation
o Staging colorectal carcinoma
 Sensitivity in detecting hepatic metastases
o Extravasation of oral or rectal contrast
 Perforation or anatomic leak
o Nonspecific findings
 Bowel wall thickening
 Mesenteric stranding
· Inflammatory bowel disease
· Enteritis/colitis
· Ischemia
o Standard CT Scan
 Insensitive for the detection of intraluminal lesions
Computed Tomography Colonography
· Virtual Colonoscopy
· Designed to overcome the limitations of conventional CT scanning
· Helical CT and 3D reconstruction to detect intraluminal colonic lesions
· Oral bowel preparation, oral and rectal contrast, colon insufflation may
maximize sensitivity
· 85-90% sensitivity and specificity in detecting 1cm or larger polyps
· Alternative for traditional colonoscopy
Magnetic Resonance Imaging
· Evaluation of pelvic lesions
· More sensitive than CT in detecting bony involvement or pelvic sidewall
extention of rectal tumors
· Accurate in detecting extent of rectal cancer spread to adjacent
mesorectum
· Reliably predict difficulty in achieving radial margin clearance of a rectal
cance
· If radial margin is threatened, neoadjuvant chemoradiation is indicated
· Helpful in detection and delineation of complex fistulas in ano
· Endorectal coil increases sensitivity
Positron Emission Tomography
· Imaging tissues with high levels of anaerobic glycolysis – malignant tumors
· F-fluorode-oxyglucose (FDG) is injected as a tracer
o Metabolism of this molecule results in positron emission
· Adjunct to CT scan in the staging of colorectal cancer
· Useful in discriminating recurrent cancer from fibrosis
· PET/CT
o Anatomic regions of high isotope accumulation (hot spots on PET)
o Increasingly used to diagnose recurrent and/or metastatic colorectal
cancer
Scintigraphy to Assess Gastrointestinal Bleeding
· Technetium 99-tagged RBC scan
· Nuclear medicine that tests Tc-erythrocytes and dynamic images to localize a
bleeding source
· Patients actively bleeding at the time of imaging
· Normal distribution of Tc-erythrocytes in vasculature, liver, spleen, penile circulation
with mild uptake in kidneys and bladder
o Can interfere with localization in bowel segments near those given structures
· Patients must be stable to tolerate imaging up to 4 hours with slow bleeding rate of
0.05-2.0mL/min
Single Photon Emission Computed Tomography (SPECT/CT)
· Radiolabeled RBCs are used
· Cross-sectional images provide a more specific location of the
bleeding source
Angiography
· Ocassionally used for detection of bleeding within the colon or small bowel
· Visualize hemorrhage
o Brisk bleeding (0.5-1.0mL/min)
· If extravasation of contrast identified, infusion of vasopression or angiographic
embolixation can be therapeutic
· If surgical resection required, angiographic catheter can be left in place to assist
in the identification of bleeding site intraoperatively
· CT and MRI angiography are also useful in assessing patency of visceral vessels
· Technique uses 3D reconstruction to detect vascular lesions
· If there is an abnormality, traditional techniques may be used for further
definition of the problem
Endorectal and Endoanal Ultrasound
· Primarily used in the evaluation of the depth of invasion
of neoplastic lesions in the rectum
· Normal rectum wall appears as a five-layer structure such
as this one
· Can reliably differentiate most benign polyp from invasive
tumors based on the integrity of the submucosal layer
· Can also differentiate superficial T1-T2 from deeper T3-
T4 tumors
· Useful in the evaluation of patients with incontinence,
constipation, rectal prolapse, obstructed defection and
other functional disorders of the pelvic floor
Manometry
· Placing a pressure-sensitive catheter in the lower rectum
· Catheter withdrawn through the anal canal and pressures recorded
· Balloon is attached to the tip of the catheter
· The resting pressure in the anal canal reflects the function of the
internal anal sphincter (N= 40-80mmHg above resting pressure)
· High pressure zone estimates lengh of the anal canal (N=2.0-4.0cm)
· The rectoanal inhibitory reflex can be detected by inflating a
balloon in the distal rectum
o Absence of this reflex = Hirschsprung Disease
Neurophysiology
· Fuction of the pudendal nerves and recruitment of puborectalis muscle fibers
· Pudenda; nerve terminal motor latency measures peed of transmission of a nerve
impulse through distal pudendal nerve fibers (N=1.8-2.2ms)
o Prolonged latency = presence of neuropathy
· EMG recruitment assesses the contraction and relaxation of the puborectalis muscels
during attempted defacation
o Recruitment increases when a patient is instructed to squeeze
o Decreases when instructed to push
o Inappropriate recruitment is an indication of paradoxical contraction
· Needle EMG has been used to map both the pudendal nerves and the anatomy of the
internal and external sphincters
· Painful and poorly tolerated by most patients
Rectal Evacuation Studies
· Balloon expulsion test and video defecography
· Balloon expulsion - Patient’s ability to expel and intrarectal balloon
· Video defecography – more detailed assessment of defecation
o Barium paste is placed in the rectum and defecation is recorded fluoroscopically
o Help diagnose
- Obstructed defecation from nonrelaxation of the puborectalis muscle or anal
sphincter dyssynergy
- Increased perineal descent
- Rectal prolapse
- Intussusception
- Rectocele
- Enterocele
o Addition of vaginal contrast and intraperitoneal contrast
4. Discuss the characteristic history
findings for benign and malignant
anal pathology
Pain
● Abdominal Pain
○ Is a nonspecific symptom with myriad causes.
○ Abdominal pain related to the colon and rectum can result from:
■ Obstruction (either inflammatory or neoplastic)
■ inflammation
■ Perforation
■ Ischemia.
○ Plain X-rays and judicious use of contrast studies and/or a CT scan can often confirm the diagnosis.
○ Gentle retrograde contrast studies (Gastrografin enema) may be useful in delineating the degree of colonic
obstruction.
○ Sigmoidoscopy and/or colonoscopy can assist in the diagnosis of ischemic colitis, infectious colitis, and
inflammatory bowel disease.
■ However, if perforation or near complete obstruction is suspected, colonoscopy and/or
sigmoidoscopy are generally contraindicated.
○ Evaluation and treatment of abdominal pain from a colorectal source should follow the usual surgical
principles of a thorough history and physical examination, appropriate diagnostic tests, resuscitation, and
appropriately timed surgical intervention.
● Pelvic Pain
○ Can originate from the distal colon and rectum or from adjacent urogenital
structures.
○ Tenesmus (is a cramping rectal pain that makes you feel that you need to have
a bowel movement, even if you already had one) may result from proctitis or
from a rectal or retrorectal mass, or fecal impaction in a constipated patient.
○ Cyclical pain associated with menses, especially when accompanied by rectal
bleeding, suggests a diagnosis of endometriosis.
○ Pelvic inflammatory disease also can produce significant abdominal and pelvic
pain.
○ The extension of a peridiverticular abscess or periappendiceal abscess into the
pelvis may also cause pain.
○ CT scan and/or MRI may be useful in differentiating these diseases.
○ Proctoscopy (if tolerated) also can be helpful.
○ Occasionally, laparoscopy will yield a diagnosis, although with access to high-
quality imaging, indications for diagnostic surgery should be rare.
Lower Gastric Bleeding
● The most common source of gastrointestinal hemorrhage is esophageal,
gastric, or duodenal, nasogastric aspiration should always be performed;
● Return of bile suggests that the source of bleeding is distal to the ligament
of Treitz.
● Colonoscopy may identify the cause of the bleeding, and cautery or
injection/application of epinephrine into the bleeding site may be used to
control hemorrhage.
● A SPECT/CT may be helpful if other modalities have not achieved
localization, particularly in distinguishing between small intestinal and
colon sources.
● Colectomy may be required if bleeding persists despite interventions.
● Intraoperative colonoscopy and/or enteroscopy may assist in localizing
bleeding.
Lower Gastric Bleeding
● Colon neoplasms bleed intermittently and rarely present with rapid
hemorrhage, the presence of occult fecal blood should always prompt a
colonoscopy.
● Unexplained iron deficiency anemia is also an indication for colonoscopy.
● Hematochezia is commonly caused by hemorrhoids or a fissure.
● Sharp, knife-like pain and bright red rectal bleeding with bowel movements
suggest the diagnosis of fissure.
● Painless, bright red rectal bleeding with bowel movements is often
secondary to a friable internal hemorrhoid that is easily detected by
anoscopy
Constipation and Obstructive Defecation
● Constipation is an extremely common complaint, affecting more than 4 million
people in the United States.
● Patients may describe infrequent bowel movements, hard stools, or excessive
straining.
● A careful history of these symptoms often clarifies the nature of the problem.
● Constipation has many causes
○ Underlying metabolic, pharmacologic, endocrine, psychological, and neurologic
causes often contribute to the problem.
● A stricture or mass lesion should be excluded by colonoscopy, barium enema, or CT
colonography.
● After these causes have been excluded, evaluation focuses on differentiating slow-
transit constipation from outlet obstruction.
Constipation and Obstructive Defecation
● Transit studies, in which radiopaque markers are swallowed and then followed
radiographically, are useful for diagnosing slow-transit constipation.
● In this study, patients ingest radiopaque studies and are followed radiographically for
5 days.
○ Retention of 20% or greater of these markers in the colon demonstrated slow
transit.
○ If these markers are congregated in the rectosigmoid colon and rectum,
obstructed defecation/outlet obstruction is suggested.
● Anorectal manometry and EMG can detect nonrelaxation of the puborectalis, which
contributes to outlet obstruction.
● The absence of an anorectal inhibitory reflex suggests Hirschsprung’s disease and may
prompt a rectal mucosal biopsy.
● Defecography can identify rectal prolapse, intussusception, rectocele, or enterocele.
Constipation and Obstructive Defecation
● Medical management is the mainstay of therapy for constipation and
includes fiber, increased fluid intake, and laxatives.
● Outlet obstruction from nonrelaxation of the puborectalis or anal sphincter
dyssynergy often responds to biofeedback.
● Surgery to correct rectocele and rectal prolapse (with or without sigmoid
resection) has a variable effect on symptoms of constipation but can be
successful in selected patients.
● Subtotal colectomy is considered only for patients with severe slow-transit
constipation (colonic inertia) refractory to maximal medical interventions.
● While this operation almost always increases bowel movement frequency,
complaints of diarrhea, incontinence, and abdominal pain are not
infrequent.
Diarrhea and Irritable Bowel Syndrome
● Diarrhea is also a common complaint and is usually a self-limited symptom of
infectious gastroenteritis.
● If diarrhea is chronic or is accompanied by bleeding or abdominal pain, further
investigation is warranted.
● Bloody diarrhea and pain are characteristic of colitis; etiology can be an infection
(invasive E coli, Shigella, Salmonella, Campylobacter, Entamoeba histolytica, or C
difficile), inflammatory bowel disease (ulcerative colitis or Crohn’s colitis), or
ischemia.
● Stool wet-mount and culture can often diagnose infection.
● Sigmoidoscopy or colonoscopy can be helpful in diagnosing inflammatory bowel
disease or ischemia.
○ However, if the patient has abdominal tenderness, particularly with peritoneal
signs, or any other evidence of perforation, endoscopy is contraindicated.
Diarrhea and Irritable Bowel Syndrome
● Chronic diarrhea may present a more difficult diagnostic dilemma.
● Chronic ulcerative colitis, Crohn’s colitis, infection, malabsorption, and
short gut syndrome can cause chronic diarrhea.
● Rarely, carcinoid syndrome and islet cell tumors (vasoactive intestinal
peptide–secreting tumor [VIPoma], somatostatinoma, gastrinoma) present
with this symptom.
● Large villous lesions may cause secretory diarrhea.
● Collagenous colitis can cause diarrhea without any obvious mucosal
abnormality.
● Along with stool cultures, tests for malabsorption, and metabolic
investigations, colonoscopy can be invaluable in differentiating these
causes.
Diarrhea and Irritable Bowel Syndrome
● Biopsies should be taken even if the colonic mucosa appears grossly normal.
● Irritable bowel syndrome is a particularly troubling constellation of
symptoms consisting of crampy abdominal pain, bloating, constipation, and
urgent diarrhea.
○ Workup reveals no underlying anatomic or physiologic abnormality.
○ Once other disorders have been excluded, dietary restrictions and avoidance of
caffeine, alcohol, and tobacco may help to alleviate symptoms.
● Antispasmodics and bulking agents may be helpful
Incontenence
● Estimated to occur in 10 to 13 individuals per 1000 people older than age 65 years.
● Incontinence ranges in severity from occasional leakage of gas and/or liquid stool to
daily loss of solid stool.
● The underlying cause of incontinence is often multifactorial, and diarrhea is often
contributory.
● In general, causes of incontinence can be classified as neurogenic or anatomic.
○ Neurogenic causes include diseases of the central nervous system and spinal cord
along with pudendal nerve injury.
○ Anatomic causes include congenital abnormalities, procidentia (rectal prolapse),
overflow incontinence secondary to impaction or an obstructing neoplasm, and
trauma.
● The most common traumatic cause of incontinence is injury to the anal sphincter
during vaginal delivery.
● Other causes include anorectal surgery, impalement, and pelvic fracture.
Incontenence
● After a thorough medical evaluation to detect underlying conditions that
might contribute to incontinence, evaluation focuses on assessment of the
anal sphincter and pudendal nerves.
● Pudendal nerve terminal motor latency testing may detect neuropathy.
● Anal manometry can detect low resting and squeeze pressures.
● Physical examination and defecography can detect rectal prolapse.
● Endoanal ultrasound is invaluable in diagnosing sphincter defects.
● Diarrhea should be treated medically (fiber, antidiarrheal agents).
● Even in the absence of frank diarrhea, the addition of dietary fiber may
improve continence.
Incontenence
● Many patients with a sphincter defect are candidates for an overlapping
sphincteroplasty.
● Sacral nerve stimulation has shown to decrease episodes of fecal
incontinence and has proven durability in the long term (5 years).
● The artificial bowel sphincter may be useful in patients who fail other
interventions.
● Other options include radiofrequency energy to the anal canal (SECCA
procedure), magnetic anal sphincter, and injectable submucosal bulking
agents, but longterm efficacy has not yet been proven.
● Finally, a stoma can provide relief for severely incontinent patients who
have failed or are not candidates for other interventions.
5. Describe indications for inpatient
and outpatient endoscopies (anoscopy
vs proctoscopy vs flexible
sigmoidoscopy and colonoscopy)
Anoscopy
● Useful instrument for examination of the
anal canal.
● A larger anoscope provides better
exposure for anal procedures such as
rubber band ligation or sclerotherapy of
hemorrhoids.
Protoscope
● Useful for examination of the rectum and
distal sigmoid colon and is occasionally
used therapeutically.
● Standard proctoscope is 25 cm in length
and available in various diameters.
● 15- or 19-mm diameter proctoscope - used
for diagnostic examinations.
● “Pediatric” proctoscope (11-mm
diameter) - better tolerated by patients
with anal stricture.
Protoscope
● Transanal Endoscopic Microsurgery (TEM)
○ Has a much wider diameter
○ Can be used for excisions of large polyps and tumors.
● Transanal minimally invasive surgery (TAMIS)
○ Can achieve similar resections to TEM
○ Does not utilize a proctoscope
○ Depends on insufflation to create a working space in the rectum while utilizing a
circular wound protector to open the anus.
Flexible Sigmoidoscopy and Colonoscopy
● Provide excellent visualization of the
colon and rectum.
● Sigmoidoscopes
○ Measure 60 cm in length.
○ Full depth of insertion may allow
visualization as high as the splenic flexure
○ Mobility and redundancy of the sigmoid
colon often limit the extent of the
examination.
○ Partial preparation with enemas is usually
adequate
○ Most patients can tolerate this procedure
without sedation
Flexible Sigmoidoscopy and Colonoscopy
● Colonoscopes
○ Measure 100 to 160 cm in length
○ Capable of examining the entire colon and terminal ileum.
○ A complete oral bowel preparation is usually necessary
○ Duration and discomfort of the procedure usually require conscious sedation.
○ Diagnostic colonoscopes
■ Possess a single channel through which instruments such as snares,
biopsy forceps, or electrocautery can be passed;
■ This channel also provides suction and irrigation capability.
○ Therapeutic colonoscopes
■ Possess two channels to allow simultaneous suction/irrigation and the
use of snares, biopsy forceps, or electrocautery.
Flexible Sigmoidoscopy and Colonoscopy
● Both sigmoidoscopy and colonoscopy can be used diagnostically and
therapeutically.
● Electrocautery should generally not be used in the absence of a complete
bowel preparation because of the risk of explosion of intestinal methane or
hydrogen gases.
6. Discuss the relevant anatomy, diagnosis and
management of a patient with
External and Internal hemorrhoids
Anal Fissure
Perirectal Abscess
Proctitis
Anal condyloma
Pilonidal disease
Fistula-in-ano
Anorectal CA
External and Internal hemorrhoids
● Hemorrhoids are cushions of submucosal tissue containing venules, arterioles, and
smooth muscle fibers that are located in the anal canal
● Three hemorrhoidal cushions are found in the left lateral, right anterior, and right
poste- rior positions. Hemorrhoids are thought to function as part of the continence
mechanism and aid in complete closure of the anal canal at rest.
● Because hemorrhoids are a normal part of anorectal anatomy, treatment is only
indicated if they become symptomatic.
● Excessive straining, increased abdominal pressure, and hard stools increase venous
engorgement of the hemorrhoidal plexus and cause prolapse of hemorrhoidal tissue.
● Bleeding, thrombosis, and symptomatic hemorrhoidal prolapse may result.
External hemorrhoids
● External hemorrhoids are located distal to the dentate line and are covered with
anoderm.
○ Because the anoderm is richly innervated, thrombosis of an external hemorrhoid
may cause significant pain.
○ It is for this reason that external hemorrhoids should not be ligated or excised
without adequate local anesthetic.
● A skin tag is redundant fibrotic skin at the anal verge, often persisting as the residua
of a thrombosed external hemorrhoid.
● Skin tags are often confused with symptomatic hemorrhoids.
● External hemorrhoids and skin tags may cause itching and difficulty with hygiene if
they are large.
● Treatment of external hemorrhoids and skin tags is only indicated for symptomatic
relief.
Internal Hemorrhoids
● Internal hemorrhoids are located proximal to the dentate line and covered by
insensate anorectal mucosa. Internal hemorrhoids may prolapse or bleed, but they
rarely become painful unless they develop thrombosis and necrosis (usually related to
severe prolapse, incarceration, and/or strangulation).
● Internal hemorrhoids are graded according to the extent of prolapse.
○ First-degree hemorrhoids bulge into the anal canal and may prolapse beyond the
dentate line on straining.
○ Second-degree hemorrhoids prolapse through the anus but reduce
spontaneously.
○ Third-degree hemorrhoids prolapse through the anal canal and require manual
reduction.
○ Fourth-degree hemorrhoids prolapse but cannot be reduced and are at risk for
strangulation.
External and Internal Hemorrhoids
● Combined internal and external hemorrhoids straddle the dentate line and have
characteristics of both internal and external hemorrhoids.
● Hemorrhoidectomy is often required for large, symptomatic, combined hemorrhoids.
● Postpartum hemorrhoids result from straining during labor, which results in edema,
thrombosis, and/or strangulation.
● Hemorrhoidectomy is often the treatment of choice, especially if the patient has had
chronic hemorrhoidal symptoms.
Risk for bleeding
● Portal hypertension was long thought to increase the risk of hemorrhoidal bleeding
because of the anastomoses between the portal venous system (middle and upper
hemorrhoidal plexuses) and the systemic venous system (inferior rectal plexuses).
○ It is now understood that hemorrhoidal disease is no more common in patients
with portal hypertension than in the normal population.
● Rectal varices, however, may occur and may cause hemorrhage in these patients.
○ In general, rectal varices are best treated by lowering portal venous pressure.
○ Rarely, suture ligation may be necessary if massive bleeding persists.
○ Surgical hemorrhoidectomy should be avoided in these patients because of the
risk of massive, difficult-to-control variceal bleeding.
Treatment
● Medical therapy
○ Bleeding from first and second-degree hemorrhoids often improves with the
addition of dietary fiber, stool softeners, increased fluid intake, and avoidance
of straining.
○ Associated pruritus often may improve with improved hygiene.
○ Many over-the-counter topical medications are desiccants and are relatively
ineffective for treating hemorrhoidal symptoms.
● Rubber Band Ligation
○ Mucosa located 1 to 2 cm proximal to the dentate line is grasped and pulled into
a rubber band applier. After firing the ligator, the rubber band strangulates the
underlying tissue, causing scarring and preventing further bleeding or prolapse
Treatment
● Infrared photocoagulation
○ Infrared photocoagulation is an effective office treatment for small first- and
second-degree hemorrhoids. The instrument is applied to the apex of each hem-
orrhoid to coagulate the underlying plexus. All three quadrants may be treated
during the same visit. Larger hemorrhoids and hemorrhoids with a significant
amount of prolapse are not effectively treated with this technique.
● Sclerotherapy
○ The injection of bleeding internal hemorrhoids with sclerosing agents is another
effective office technique for treatment of first-, second-, and some third-
degree hemorrhoids. One to 3 mL of a sclerosing solution (phenol in olive oil,
sodium morrhuate, or quinine urea) is injected into the submucosa of each
hemorrhoid. Few complications are associated with sclerotherapy, but infection
and fibrosis have been reported.
Treatment
● Excision of Thrombosed External Hemorrhoids
○ Acutely thrombosed external hemorrhoids generally cause intense pain and a
palpable perianal mass during the first 24 to 72 hours after thrombosis.
○ The thrombosis can be effectively treated with an elliptical excision performed
in the office under local anesthesia.
○ Because the clot is usually loculated, simple incision and drainage is rarely
effective. After 72 hours, the clot begins to resorb, and the pain resolves
spontaneously.
○ Excision is unnecessary, but sitz baths and analgesics are often helpful.
● Operative Hemorrhoidectomy
○ A number of surgical procedures have been described for elective resection of
symptomatic hemorrhoids.
○ All are based on decreasing blood flow to the hemorrhoidal plexuses and excising
redundant anoderm and mucosa.
Treatment
● Closed Submucosal Hemorrhoidectomy
○ The Parks or Fer-guson hemorrhoidectomy involves resection of hemorrhoidal tissue and closure
of the wounds with absorbable suture.
○ The procedure may be performed in the prone or lithotomy position under local, regional, or
general anesthesia.
○ The anal canal is examined and an anal speculum inserted.
○ The hemorrhoid cushions and associated redundant mucosa are identified and excised using an
elliptical incision starting just distal to the anal verge and extending proximally to the
anorectal ring.
○ It is crucial to identify the fibers of the internal sphincter and carefully brush these away from
the dissection in order to avoid injury to the sphincter.
○ The apex of the hemorrhoidal plexus is then ligated and the hemorrhoid excised.
○ The wound is then closed with a running absorbable suture.
○ All three hemorrhoidal cushions may be removed using this technique;
■ however, care should be taken to avoid resecting a large area of perianal skin in order to
avoid postoperative anal stenosis
Treatment
● Open Hemorrhoidectomy
○ This technique, often called the Milligan and Morgan hemorrhoidectomy, follows
the same principles of excision described earlier, but the wounds are left open
and allowed to heal by secondary intention.
● Whitehead’s Hemorrhoidectomy
○ Whitehead’s hemorrhoidectomy involves circumferential excision of the
hemorrhoidal cushions just proximal to the dentate line.
○ After excision, the rectal mucosa is then advanced and sutured to the dentate
line
○ Risk of ectropion (Whitehead’s deformity).
Treatment
● Procedure for Prolapse and Hemorrhoids/Stapled Hemorrhoidectomy
○ Procedure for prolapse and hemorrhoids (PPH) is also referred to as a stapled
hemorrhoidopexy.
○ Best suited for patients with second and third-degree hemorrhoids, this out-
patient procedure uses a stapling device similar in appearance and mechanism
of action to an end-to-end anastomotic (EEA) stapling device used for rectal
surgery.
○ Just as with an EEA stapler, proximal and distal tissue donuts, in this case
consisting of mucosa and submucosa, are generated by the PPH stapler though
the primary means by which this procedure provides relief for internal
hemorrhoids is by pexying the redundant hemorrhoidal tissue, ligating the
venules feeding the hemorrhoidal plexus and fixing redundant mucosa proximal
to the dentate line.
○ Several studies suggest that this procedure is safe and effective, that it is
associated with less postoperative pain and disability, and that it has an
equivalent risk of postoperative complications when compared to excisional
hemorrhoidectomy.
Anal Fissure
● A fissure in ano is a tear in the anoderm distal to the dentate line.
● The pathophysiology of anal fissure is thought to be related to trauma from either the
passage of hard stool or prolonged diarrhea.
● A tear in the anoderm causes spasm of the internal anal sphincter, which results in
pain, increased tearing, and decreased blood supply to the anoderm.
○ This cycle of pain, spasm, and ischemia contributes to development of a poorly
healing wound that becomes a chronic fissure.
○ The vast majority of anal fissures occur in the posterior midline.
○ Ten percent to 15% occur in the anterior midline. Less than 1% of fissures occur
off midline
Anal Fissure
● Anal fissure is extremely common.
● Characteristic symptoms include tearing pain with defecation and hematochezia
(usually described as blood on the toilet paper).
● Patients may also complain of a sensation of intense and painful anal spasm lasting
for several hours after a bowel movement.
● On physical examination, the fissure can often be seen in the anoderm by gently
separating the buttocks.
● Patients are often too tender to tolerate digital rectal examination, anoscopy, or
proctoscopy.
● An acute fissure is a superficial tear of the distal anoderm and almost always heals
with medical management.
Anal Fissure
● Chronic fissures develop ulceration and heaped-up edges with the white fibers of the
internal anal sphincter visible at the base of the ulcer.
● There often is an associated external skin tag and/or a hypertrophied anal papilla
internally.
● These fissures are more challenging to treat and may require surgery.
● A lateral location of a chronic anal fissure may be evidence of an underlying disease
such as Crohn’s disease, HIV, syphilis, tuberculosis, or leukemia.
● If the diagnosis is in doubt or there is suspicion of another cause for the perianal pain
such as abscess or fistula, an examination under anesthesia may be necessary.
Treatment
○ Therapy focuses on breaking the cycle of pain, spasm, and ischemia thought to be
responsible for development of fissure in ano.
○ First-line therapy to minimize anal trauma includes bulk agents, stool softeners, and
warm sitz baths.
○ The addition of 2% lidocaine jelly or other analgesic creams can provide additional
symptomatic relief.
○ Nitroglycerin ointment has been used locally to improve blood flow but often causes
severe headaches.
○ Both oral and topical calcium channel blockers (diltiazem and nifedipine) have also
been used to heal fissures and may have fewer side effects than topical nitrates.
○ Newer agents, such as arginine (a nitric oxide donor) and topical bethanechol (a
muscarinic agonist), have also been used to treat fissures.
○ Medical therapy is effective in most acute fissures, but it will heal only approximately
50% of chronic fissures.
● Botulinum toxin (Botox) causes temporary muscle paralysis by preventing acetylcholine
release from presynaptic nerve terminals.
● Injection of botulinum toxin is used in some centers as an alternative to surgical
sphincterotomy for chronic fissure.
● Although there are few long-term complications from the use of botulinum toxin, healing
appears to be equivalent to other medical therapies.
● Surgical therapy has traditionally been recommended for chronic fissures that have failed
medical therapy, and lateral internal sphincterotomy is the procedure of choice.
● The aim of this procedure is to decrease spasm of the internal sphincter by dividing a
portion of the muscle.
● Approximately 30% of the internal sphincter fibers are divided laterally by using either an
open or closed technique.
● Healing is achieved in more than 95% of patients using this technique, and most patients
experience immediate pain relief.
● Recurrence occurs in less than 10% of patients, and the risk of incontinence (usually to
flatus) ranges from 5% to 15%.
● Advancement flaps (VY) with or without sphincterotomy have also been reported to
successfully treat chronic fissures.
Perirectal Abscess
● The majority of anorectal suppurative disease results from infections of the anal
glands (cryptoglandular infection) found in the intersphincteric plane.
● Their ducts traverse the internal sphincter and empty into the anal crypts at the level
of the dentate line.
● Infection of an anal gland results in the formation of an abscess that enlarges and
spreads along one of several planes in the perianal and perirectal spaces.
● The perianal space surrounds the anus and laterally becomes continuous with the fat
of the buttocks.
● The intersphincteric space separates the internal and external anal sphincters.
● It is continuous with the perianal space distally and extends cephalad into the rectal
wall.
● The ischiorectal space (ischiorectal fossa) is located lateral and posterior to the anus
and is bounded medially by the external sphincter, laterally by the ischium, superiorly
by the levator ani, and inferiorly by the transverse septum.
● The ischiorectal space contains the inferior rectal vessels and lymphatics.
● The two ischiorectal spaces connect posteriorly above the anococcygeal ligament but
below the levator ani muscle, forming the deep postanal space.
● The supralevator spaces lie above the levator ani on either side of the rectum and
communicate posteriorly.
● The anatomy of these spaces influences the location and spread of cryptoglandular
infection
● As an abscess enlarges, it spreads in one of several directions.
● A perianal abscess is the most common manifestation and appears as a painful
swelling at the anal verge. Spread through the external sphincter below the level of
the puborectalis produces an ischiorectal abscess.
● These abscesses may become extremely large and may not be visible in the perianal
region.
● Digital rectal exam will reveal a painful swelling laterally in the ischiorectal fossa.
Intersphincteric abscesses occur in the intersphincteric space and are notoriously
difficult to diagnose, often requiring an examination under anesthesia.
● Pelvic and supralevator abscesses are uncommon and may result from extension of an
intersphincteric or ischiorectal abscess upward or extension of an intraperitoneal
abscess downward.
● Diagnosis
● Severe anal pain is the most common presenting complaint.
● A palpable mass is often detected by inspection of the perianal area or by digital
rectal examination. Occasionally, patients will present with fever, urinary retention,
or life-threatening sepsis.
Diagnosis and Treatment
● The diagnosis of a perianal or ischiorectal abscess can usually be made with physical
exam alone (either in the office or in the operating room). However, complex or
atypical presentations may require imaging studies such as CT or MRI to fully
delineate the anatomy of the abscess.
● Anorectal abscesses should be treated by drainage as soon as the diagnosis is
established.
○ If the diagnosis is in question, an examination and drainage under anesthesia are
often the most expeditious ways both to confirm the diagnosis and to treat the
problem.
○ Delayed or inadequate treatment may occasionally cause extensive and life-
threatening suppuration with massive tissue necrosis and septicemia.
○ Antibiotics are only indicated if there is extensive overlying cellulitis or if the
patient is immunocompromised, has diabetes mellitus, or has valvular heart
disease.
○ Antibiotics alone are ineffective at treating perianal or perirectal infection.
Perianal abscess
● Most perianal abscesses can be drained under local anesthesia in the office, clinic, or
emergency department.
● Larger, more complicated abscesses may require drainage in the operating room.
● A skin incision is created, and a disk of skin excised to prevent premature closure.
● No packing is necessary, and sitz baths are started the next day
Intersphincteric abscess
● Intersphincteric abscesses are notoriously difficult to diagnose because they produce little
swelling and few perianal signs of infection.
● Pain is typically described as being deep and “up inside” the anal area and is usually
exacerbated by coughing or sneezing.
● The pain is so intense that it usually precludes a digital rectal examination.
● The diagnosis is made based on a high index of suspicion and usually requires an
examination under anesthesia.
● Once identified, an intersphincteric abscess can be drained through a limited, usually
posterior, internal sphincterotomy.
Ischiorectal abscess
● An ischiorectal abscess causes diffuse swelling in the ischiorectal fossa
that may involve one or both sides, forming a “horseshoe” abscess.
● Simple ischiorectal abscesses are drained through an incision in the
overlying skin.
● Horseshoe abscesses require drainage of the deep postanal space and
often require counter incisions over one or both ischiorectal spaces
Supralevator Abscess
● This type of abscess is uncommon and can be difficult to diagnose.
○ Because of its proximity to the peritoneal cavity, supralevator abscesses can
mimic intra-abdominal conditions.
● Digital rectal examination may reveal an indurated, bulging mass above the anorectal
ring.
● It is essential to identify the origin of a supralevator abscess prior to treatment.
● If the abscess is secondary to an upward extension of an intersphincteric abscess, it
should be drained through the rectum.
● If it is drained through the ischiorectal fossa, a complicated, suprasphincteric fistula
may result.
● If a supralevator abscess arises from the upward extension of an ischiorectal abscess,
it should be drained through the ischiorectal fossa.
● Drainage of this type of abscess through the rectum may result in an extrasphincteric
fistula.
● If the abscess is secondary to intra-abdominal disease, the primary process requires
treatment and the abscess is drained via the most direct route (transabdominally,
rectally, or through the ischiorectal fossa).
Proctitis
● Bacterial Infections
○ Proctitis is a common symptom of anorectal bacterial infection.
○ Neisseria gonorrhoeae is the most common bacterial cause of proctitis and causes pain,
tenesmus, rectal bleeding, and mucus discharge.
○ Chlamydia trachomatis infection may be asymptomatic or may produce similar symptoms.
○ Treponema pallidum, the microbe causing syphilis, causes a chancre at the site of inoculation,
which may be asymptomatic or may present as an atypical fissure (primary syphilis).
○ Condyloma lata are characteristic of secondary syphilis.
○ Chancroid, caused by Haemophilus ducreyi, is a disease manifested by multiple painful,
bleeding lesions.
○ Inguinal lymphadenopathy and fluctuant, draining lymph nodes are characteristic. Donovania
granulomatis infection produces shiny, red masses on the perineum (granuloma inguinale).
○ Diarrheal illnesses caused by organisms such as Campylobacter or Shigella may also be sexually
transmitted.
○ Treatment consists of antimicrobial agents directed against the infecting organism.
Proctitis
● Parasitic Infections
○ Entamoeba histolytica is an increasingly common sexually transmitted disease.
○ Amoebas produce ulcerations in the gastrointestinal mucosa and can infect any part of
the gut.
○ Symptoms include diarrhea, abdominal pain, and tenesmus.
○ Giardia lamblia is also common and produces diarrhea, abdominal pain, and malaise.
● Viral Infections
○ Herpes proctitis is extremely common.
○ Proctitis is usually caused by type 2 herpes simplex virus and less commonly by type 1
herpes simplex virus.
○ Patients complain of severe, intractable perianal pain and tenesmus.
○ Pain often precedes the development of characteristic vesicles, and these patients
may require an examination under anesthesia to exclude another diagnosis such as an
intersphincteric abscess.
○ Diagnosis is confirmed by viral culture of tissue or vesicular fluid.
Anal condyloma
● Condylomas occur in the perianal area or in the squamous
epithelium of the anal canal.
● Occasionally, the mucosa of the lower rectum may be affected
Anal condyloma
● HPV causes condyloma acuminata (anogenital warts) and is associated with squamous
intraepithelial lesions and squamous cell carcinoma
● HPV types 6 and 11 commonly cause warts, but do not appear to cause malignant degeneration
● Treatment
○ Small warts on the perianal skin and distal anal canal
■ may be treated in the office with topical application of bichloracetic acid or podophyllin.
● Although 60% to 80% of patients will respond to these agents, recurrence and reinfection are
common.
○ Imiquimod (Aldara): immunomodulator
■ highly effective in treating condyloma located on the perianal skin and distal anal canal.
○ Larger and/ or more numerous warts
■ excision and/or fulguration in the operating room.
■ Excised warts should be sent for pathologic examination to rule out dysplasia or
malignancy.
○ The recent introduction of a vaccine against HPV holds promise for preventing
anogenital condylomas
Pilonidal disease
● it is speculated that the cleft creates a suction that draws hair into the
midline pits when a patient sits.
● These ingrown hairs may then become infected and present acutely as an
abscess in the sacrococcygeal region
Pilonidal disease
● Once an acute episode has resolved, recurrence is common.
● An acute abscess should be incised and drained as soon as the diagnosis
is made.
● Because these abscesses are usually very superficial, this procedure can
often be performed in the office, clinic, or emergency department
under local anesthetic.
● Because midline wounds in the region heal poorly, some surgeons
recommend using an incision lateral to the intergluteal cleft.
Pilonidal disease
● Treatment
○ The simplest method involves unroofing the tract, curetting the base, and
marsupializing the wound.
○ The wound must then be kept clean and free of hair until healing is complete
(often requiring weekly office visits for wound care).
○ Alternatively, a small lateral incision can be created and the pit excised.
■ effective for most primary pilonidal sinuses.
○ Complex and/or recurrent sinus tracts
■ require more extensive resection and closure with a Z-plasty, advancement
flap, or rotational flap
Fistula-in-ano
● The fistula usually originates in the infected crypt
(internal opening) and tracks to the external
opening, usually the site of prior drainage
Fistula-in-ano
Diagnosis
● Patients present with persistent drainage
from the internal and/or external openings
● An indurated tract is often palpable.
● external opening is often easily identifiable
● identification of the internal opening may
be more challenging.
● Goodsall’s rule can be used as a guide in
determining the location of the internal
opening
Fistula-in-ano
● fistulas with an external opening anteriorly connect to the internal opening by a short, radial tract.
● Fistulas with an external opening posteriorly track in a curvilinear fashion to the posterior midline
● Categories:
○ intersphincteric fistula
■ tracks through the distal internal sphincter and intersphincteric space to an external opening near
the anal verge
○ transsphincteric fistula
■ results from an ischiorectal abscess and extends through both the internal and external
sphincters
○ suprasphincteric fistula
■ originates in the intersphincteric plane and tracks up and around the entire external sphincter
○ extrasphincteric fistula
■ originates in the rectal wall and tracks around both sphincters to exit laterally, usually in the
ischiorectal fossa
Fistula-in-ano
Treatment
● surgical treatment is dictated by the location of the internal and external openings and the
course of the fistula.
● Simple intersphincteric fistulas
○ can often be treated by fistulotomy (opening the fistulous tract), curettage, and
healing by secondary intention
● transsphincteric fistula
○ Fistulas that include less than 30% of the sphincter muscles
■ treated by sphincterotomy
○ High transsphincteric fistulas, which encircle a greater amount of muscle
■ more safely treated by initial placement of a seton.
● suprasphincteric fistulas
○ treated with seton placement
● Extrasphincteric fistulas
○ treatment depends on both the anatomy of the fistula and its etiology
Fistula-in-ano
● Complex fistulas with multiple tracts may require numerous procedures to control
sepsis and facilitate healing.
● Failure to heal may ultimately require fecal diversion
● Proctoscopy
○ performed in all cases of complex and/or nonhealing fistulas to assess the health of the rectal
mucosa.
● Biopsies of the fistula tract should be taken to rule out malignancy.
● Higher fistulas may be treated by an endorectal advancement flap
Anorectal CA
● Neoplasms of the anal canal have traditionally been divided into those
affecting the anal margin (distal to the dentate line) and those affecting the
anal canal (proximal to the dentate line) based on lymphatic drainage patterns.
● Lymphatics from the anal canal proximal to the dentate line drain cephalad via
the superior rectal lymphatics to the inferior mesenteric nodes and laterally
along both the middle rectal vessels and inferior rectal vessels through the
ischiorectal fossa to the internal iliac nodes.
● Lymph from the anal canal distal to the dentate line usually drains to the
inguinal nodes.
● It can also drain to the superior rectal lymph nodes or along the inferior rectal
lymphatics to the ischiorectal fossa if primary drainage routes are blocked with
tumor
Anorectal CA
Squamous intraepithelial lesions
● High-grade lesions are precursors to invasive squamous cell carcinoma (epidermoid
carcinoma) and may appear as a plaque or may only be apparent with high-resolution
anoscopy and application of acetic acid and/or Lugol’s iodine solution.
● The incidence of both squamous intraepithelial lesions and epidermoid carcinoma of
the anus has increased dramatically among human immunodeficiency virus (HIV)–
positive men who have sex with men.
● This increase is thought to result from increased rates of HPV infection along with
HIV-induced immunosuppression
Anorectal CA
● Treatment
○ ablation
■ high-grade dysplasia
■ Because of a high recurrence and/or reinfection rate, these patients require close
surveillance.
○ High-risk patients should be followed with frequent anal Papanicolaou (Pap) smears every 3 to 6
months.
■ abnormal Pap smear should be followed by an examination under anesthesia and anal
mapping using highresolution anoscopy.
○ High-resolution anoscopy shows areas with abnormal telangiectasias that are consistent with high
grade dysplasia.
○ Medical therapy for HPV has also been proposed.
■ Topical immunomodulators such as imiquimod (Aldara) have been shown to induce regression
in some series.
■ Topical 5-fluorouracil has also been used in this setting.
○ Finally, the introduction of a vaccine against HPV may help decrease the incidence of this disease
in the future
Anorectal CA
Epidermoid Carcinoma.
● slow-growing tumor and usually presents as an intra-anal or perianal mass.
● Pain and bleeding may be present.
● Treatment
○ Perianal epidermoid carcinoma may be treated in a similar fashion as squamous
cell carcinoma of the skin in other locations because wide local excision can usually
be achieved without resecting the anal sphincter.
○ Intra-anal epidermoid carcinoma cannot be excised locally
■ first-line therapy relies on chemotherapy and radiation (the Nigro protocol: 5-
fluorouracil, mitomycin C, and 30 Gy of external beam radiation).
○ It is important to note that epidermoid carcinomas continue to respond after
completion of chemoradiation.
○ Lesions that persist greater than 3 to 6 months after therapy may represent persistent
disease and should be biopsied.
○ Recurrence: requires radical resection (APR).
○ Metastasis to inguinal lymph nodes is a poor prognostic sign
Anorectal CA
Verrucous Carcinoma (Buschke-Lowenstein Tumor, Giant Condyloma Acuminata).
● locally aggressive form of condyloma acuminata.
● Although these lesions do not metastasize, they can cause extensive local tissue
destruction
● may be grossly indistinguishable from epidermoid carcinoma.
● Treatment
○ Wide local excision (treatment of choice)
○ radical resection
○ Topical immunomodulators such as imiquimod (Aldara)
○ may shrink some tumors, but they are almost never curative.
○ Very large lesions: external beam radiation, but resection is almost always
required.
Anorectal CA
Basal Cell Carcinoma
● rare and resembles basal cell carcinoma elsewhere on the skin
(raised, pearly edges with central ulceration).
● slowgrowing tumor that rarely metastasizes.
● Treatment
○ Wide local excision (treatment of choice)
■ recurrence occurs in up to 30% of patients.
○ Radical resection and/or radiation therapy
■ for large lesions
Anorectal CA
Adenocarcinoma
● extremely rare and usually represents downward spread of a low rectal adenocarcinoma
● may occasionally arise from the anal glands or may develop in a chronic fistula.
● Treatment
○ Radical resection, usually after neoadjuvant chemoradiation
● Extramammary perianal Paget’s disease
○ is adenocarcinoma in situ arising from the apocrine glands of the perianal area
○ typically plaque-like and may be indistinguishable from high-grade intraepithelial
lesions.
○ Histology: Paget’s cells are seen.
○ Treatment
■ These tumors are often associated with a synchronous gastrointestinal adenocarcinoma,
so a complete evaluation of the intestinal tract should be performed.
■ Wide local excision
● for perianal Paget’s disease
Anorectal CA
Melanoma
● Anorectal melanoma is rare
○ < 1% of all anorectal malignancies
○ 1% to 2% of melanomas.
● Treatment
○ prognosis for patients with anorectal disease: poor.
■ Overall 5-year survival: <10%
○ A few patients with anorectal melanoma
■ present with isolated local or locoregional disease that is potentially resectable for cure,
and both radical resection (APR) and wide local excision have been advocated.
○ Recurrence is common
○ wide local excision: for initial treatment of localized anal melanoma
○ The addition of adjuvant chemotherapy, biochemotherapy, vaccines, or radiotherapy
may be of benefit in some patients, but efficacy remains unproven
Case 1: A 33 year-old female comes to see you complaining
of perirectal pain
1) Differential diagnoses
B) What pertinent questions will you ask during
history taking? Why?
• Pain
 How long has it been present?
 Is it constant?
 What makes it better or worse?
 Is the pain increasing or decreasing?
 What is the quality of the pain?
 Patients with thrombosed or incarcerated (non-reducible prolapsed) hemorrhoids usually present with severe, constant pain that has
come on suddenly. Often, patients will recall an episode of severe constipation or lifting heavy objects that preceded the pain.
 Another diagnosis that has a characteristic pain quality is an anal fissure. These symptoms also
often present after an episode of severe constipation or anal trauma, and are characterized as a
very sharp, cutting or tearing pain, often described as “passing glass” during defecation or having
“a sharp knife poking” the anus. They may also complain of burning for hours after. Often,
patients with a fissure note that prior to their bowel movement, they did not have pain.
 Pain that is constant but comes on gradually over the course of several days is characteristic of a
perianal or perirectal abscess or an anal sexually transmitted disease (i.e., syphilis or herpes).
 Pain that worsens over many weeks or months is typical of proctitis and malignancies.
 In general, moderate or mild hemorrhoidal disease is not associated with significant pain, though
patients may report some discomfort or itching in the area.
• Presence of mass?
 Is there a mass or swelling noted by the patient?
 Is it new?
 Is it enlarging?
 Is it always present or does it at times disappear?
 Is there more than one mass?
 Patients with the most common types of perirectal abscess, pilonidal abscess, and thrombosed external or
incarcerated hemorrhoids will appreciate a new mass or swelling which is tender to touch.
 Anal fissures can be associated with an anal skin tag (also known as a sentinel pile) that patients may
notice.
 Patients with intermittent grade II or grade III hemorrhoids can have protrusion of tissue.
 Patients with anal condyloma can also note new masses, which tend to be small and multiple.
 Other more concerning things can also present as a new mass, including anal cancers
 Less commonly, rectal prolapse can also present as a new large mass that can be confused with
hemorrhoids
• Presence of bleeding?
 How much?
 What is the location of the blood: on the toilet paper? In the toilet water?
 On top of the stool, or mixed in with the stool?
 Are there symptoms of anemia?
 Internal hemorrhoids classically bleed with bowel movements, resulting in blood on the tissue or in the
toilet water and coating the stools. Sometimes, the bleeding can be severe enough to cause anemia,
though generally, it is mild.
 Anal fissures also have a similar bleeding pattern, though these are often associated with pain.
• Proctitis patients classically will have urgency and frequency and often have frequent, small, bloody
bowel movements. The bleeding can be bright or darker red. Bleeding can occur with pilonidal disease if
there is a break in the skin, though the bleeding is mild and usually the patient can appreciate the
bleeding is not near the anus but at the top of the gluteal crease, not related to bowel movements and
frequently just located on the underwear.
 Thrombosed external hemorrhoids may have mild bleeding seen on the toilet paper or in the
underwear.
 Malignancies often bleed with even gentle touch or manipulation.
 Occasionally, if a perianal abscess has developed into a perianal fistula, the external opening of the
fistula can also have mild bleeding.
• Presence of drainage?
 How much?
 What is the character?
 The classic draining lesion in the perianal region is a perianal fistula, which produces scant, thick yellow or
greenish-tinged discharge.
 Abscesses that have spontaneously opened can produce some drainage, which is usually copious at first
and rapidly decreases in volume.
 Prolapsed internal hemorrhoids or rectal prolapse can also produce some drainage, though this tends to be
thin, white or clear drainage and occasionally pink-tinged
• Other questions?
 There are other important questions to ask for patients with any perianal problem, including inquiring
about abdominal pain and any change in their bowel habits, including what they consider normal and
regular.
 Additionally, soft but formed stool is the ideal, which should require little to no straining to evacuate.
• It is also important to inquire about the patients’ control of their bowels and any accidents and
leakage they may have, as fecal incontinence is a frequent problem in older women due to previous
obstetrical injuries during vaginal deliveries
C) How do you do digital rectal exam?
•Introduction
•Introduce yourself with your name, role and confirm the patient’s name and date of birth. Briefly
explain the procedure and the degree of exposure required. The patient should be undressed from
the waist down including any undergarments.
•Obtain consent and proceed to wash hands. For this examination, it is essential to have a
chaperone.
•Position the bed flat and then put on a pair of non-sterile gloves and an apron. Instruct the patient
to lie on the bed in the left lateral position with their knees bent up to their chest. Offer the patient
a blanket to maintain their dignity. Ask if they are in any pain and are comfortable.
•Equipment
•Before you begin your examination, prepare the following equipment in a plastic tray:
 Non-sterile gloves.
 Disposable apron.
 Tissues.
 Lubricating jelly.
•Inspection
•Warn the patient that you are about to look around the area.
•Separate the buttocks using your non-dominant hand and inspect the anus and natal cleft. Inspect for the following signs:
 External haemorrhoids
 Skin tags (fleshy extra skin hanging from anus): These are usually harmless growths that may be mistaken for warts or haemorrhoids.
They can be caused by inflammation, injury or leftover skin after haemorrhoid removal.
 Excoriation marks (scratch marks): These are as a result of anal itchiness, which can be due infection, haemorrhoids, bowel
incontinence, long-term constipation/diarrhoea or repeated glyceryl trinitrate cream topical application (used to treat anal fissures).
 Anal fissures (skin tear): These occur with long term straining of hard faeces such as with chronic constipation or in inflammatory
bowel disease.
 Anal fistulae (small opening often with discharge): These can also be present in inflammatory bowel disease (particularly Crohn’s
disease). Other causes include abscesses that have failed to heal and diverticulitis.
 Ask the patient to cough and check for any internal haemorrhoids or rectal prolapse.
•Anal tone
•Lubricate the index finger of your dominant hand with some lubricating jelly and place your other
hand on the patient’s hip. Place one finger on the anal orifice and lightly palpate for induration (a
hardened/fibrous section under the skin), which is associated with inflammatory conditions such as
Crohn’s disease.
•Ask the patient to breathe in deeply and then relax to loosen the sphincter.
•Warn the patient that they may feel some cold jelly, and then advance your index finger into the
anal canal. Ask the patient to bear down on your finger to assess anal tone. Decreased anal tone
may be due to long-term diarrhoea, diabetes, spinal trauma or simply old age.
Palpation
•Anal canal
•Sweep 360° around the full rectum. The anterior walls may be more easily examined by rotating
your body and wrist. Check for any lumps; these may be haemorrhoids, polyps or a tumour. Feel for
palpable faeces and tenderness. If the patient is in severe pain during the examination, consider an
anal fissure, abscess or ulcer.
•Note the location, using a clock face, and texture of anything you find. e.g. 1cm irregular mass at 9
o’clock.
•After advancing your finger, it may be worth pausing for a few seconds to allow the patient to
acclimatise and relax.
•During palpation look at the patient’s face for signs of distress/pain.
•Prostate
•With males, identify the prostate, its central sulcus and assess the size. A normal prostate is
smooth and walnut sized. Check for symmetry, the presence of any nodules and any tenderness.
Abnormalities in these features can give indications towards pathology:
 Enlarged with a deep sulcus: Benign prostatic hyperplasia.
 Enlarged and very tender: Prostatitis.
 Enlarged with a rough nodule: Unilateral cancer.
 Hard, asymmetrical and irregular with an impalpable sulcus: Prostate cancer.
•Completion
•Inspection of the gloved finger
•Withdraw and inspect the gloved finger for blood or mucus, suggesting ulcerative colitis. Thank the
patient, remove your gloves and place them in a clinical waste bin. Wash your hands.
•Wipe away the jelly from the anus and offer the patient some extra tissues for their own comfort.
Give the patient privacy to get dressed.
•Post-examination
•Offer to take a focused history, an abdominal exam and the appropriate tests depending on the
pathology.
D) What will you request for further
diagnostics? And why?
1) Anoscopic examination: Most specific and conclusive diagnostic test for hemorrhoids
 An anoscope is approximately 7cm long. It comes in various sizes and shapes. This does not require bowel
prep, nor does it require sedation, as this can be easily done in the office for most patients.
 If a patient has significant pain on exam, and a cause for the pain cannot be determined in the office, then
an exam with sedation can be done in the GI lab or in the operating room, if needed.
 The anoscope allows one to see the whole anal canal and, depending on patient habitus and type of
anoscope used, it can also allow one to see the distal rectum for 2- 4cm above the dentate line.
 It is helpful to evaluate internal hemorrhoids, the extent of a small anal cancer or anal condyloma within the
anal canal, as well as to look for internal fistula openings.
2) Colonoscopy/flexible sigmoidoscopy: Usually normal in hemorrhoids;
may reveal other pathologies
 Used to exclude serious pathology such as inflammatory bowel disease or cancer
 In presence of suspicious symptoms such as altered bowel habit (diarrhea and/or constipation),
abdominal pain, weight loss, iron deficiency anemia or passage of blood clots and/or mucus, lower
gastrointestinal endoscopy is performed.
 Definitive test is colonoscopy
• If flexible sigmoidoscopy is chosen, it should be combined with barium enema to assess proximal colon
in high risk patients(eg: family history of bowel cancer)
3) PROCTOSCOPY: A proctoscope may also be referred to as a rigid sigmoidoscope or rigid proctoscope
and is 25cm long. As the average rectum is 15cm long, it is obvious that with full insertion,
 the lower part of the sigmoid can be seen.
 This does not require a bowel prep, but for best evaluation, a patient will perform one or two enemas
prior to the procedure to allow the rectum to be free from stool. This also does not require sedation if the
patient does not have severe pain or anxiety, and can be routinely done in the office.
 Patients may be positioned in the knee-chest position but more frequently, they are either positioned in a
lateral decubitus position or on a procto table.
 This is frequently performed to evaluate malignancies that may be extending more proximal than what
can be seen by an anoscope.
 This is the standard technique used to measure the distal edge of a higher tumor from the anal verge,
as is done for rectal cancers, to determine the location in the rectum.
 The rigid scope allows a straight measurement to be taken, unlike a flexible scope that can lead to
inaccuracies due to looping or flexing of the scope. Since the entire rectum can be visualized, this is an
ideal scope to evaluate for proctitis and to perform biopsies of any lesions in the rectum
• 4) FBC:Ordered only if there is concern that the patient has experienced significant
prolonged rectal bleeding and signs of anemia are present
• 5) Stool for occult blood: Unnecessary unless no significant
hemorrhoidal tissue is seen on examination; Further evaluation deemed unnecessary if the
results are negative
E) How will you manage this patient? (Based
on differential diagnosis)
• HEMORRHOIDS TREATMENT
 Medical Therapy
 improves with the addition of dietary fiber, stool softeners, increased fluid intake, and avoidance of straining.
 pruritus often may improve with improved hygiene.
 Rubber Band Ligation .
 first-, second-, and selected third-degree hemorrhoids may be treated by rubber band ligation Mucosa located 1
to 2 cm proximal to the dentate line is grasped and pulled into a rubber band applier. After firing the ligator, the
rubber band strangulates the underlying tissue, causing scarring and preventing further bleeding or prolapse
 Infrared Photocoagulation
 treatment for small first- and second-degree hemorrhoids.
 The instrument is applied to the apex of each hemorrhoid to coagulate the underlying plexus.Larger hemorrhoids and with a significant amount of
prolapse are not effectively treated with this technique.
 Sclerotherapy The injection of bleeding internal hemorrhoids with sclerosing agents is another effective office technique for treatment of first-, second-,
and some third-degree hemorrhoids.
 One to 3 mL of a sclerosing solution (phenol in olive oil, sodium morrhuate, or quinine urea) is injected into the submucosa of each hemorrhoid.
 Excision of Thrombosed External Hemorrhoids Acutely thrombosed external hemorrhoids- cause intense pain and a palpable perianal mass during the
first 24 to 72 hours after thrombosis.
 The thrombosis - elliptical excision performed in the office under local anesthesia. Because the clot is usually loculated, simple incision and drainage is
rarely effective
 . After 72 hours, the clot begins to resorb, and the pain resolves spontaneously.
 Excision is unnecessary, but sitz baths and analgesics are often helpful.
 Operative Hemorrhoidectomy
 Closed Submucosal Hemorrhoidectomy
 the Parks or Ferguson hemorrhoidectomy involves resection of hemorrhoidal tissue and
closure of the wounds with absorbable suture.
 Open Hemorrhoidectomy This technique, often called the Milligan and Morgan
hemorrhoidectomy, follows the same principles of excision described earlier, but the wounds
are left open and allowed to heal by secondary intention.
• Whitehead’s Hemorrhoidectomy involves circumferential excision of the hemorrhoidal cushions
just proximal to the dentate line. After excision, the rectal mucosa is then advanced and sutured to
the dentate line. risk of ectropion (Whitehead’s deformity).
 Anal fissure: First-line therapy -bulk agents, stool softeners, and warm sitz baths.
The 2% lidocaine jelly or other analgesic creams can provide symptomatic relief.
 Nitroglycerin ointment -improve blood flow
 Both oral and topical calcium channel blockers (diltiazem and nifedipine) have also been
used to heal fissures
 Botulinum toxin (Botox) causes temporary muscle paralysis. It is an alternative to surgical
sphincterotomy for chronic fissure.
 Surgical therapy -lateral internal sphincterotomy The aim of this procedure is to
decrease spasm of the internal sphincter -open or closed technique.
 Rectovaginal fistula: The treatment of rectovaginal fistula depends on the size, location, etiology, and condition of
surrounding tissues.
 Because up to 50% of fistulas caused by obstetric injury heal spontaneously
 Low and mid-rectovaginal fistulas are usually best treated with an endorectal advancement flap
 Fistulas caused by malignancy should be treated with resection of the tumor
 Perianal abscess: Most perianal abscesses can be Treated By draining under local anesthesia in the office, clinic, or
emergency department.
 Larger, more complicated abscesses may require drainage in the operating room
 A skin incision is created, and a disk of skin excised to prevent premature closure
Case 2: A 63 year old woman presents to
the office for evaluation of a painful
anal mass
A) What pertinent questions will you ask
during history taking? Why?
• HISTORY OF PRESENT ILLNESS
 Onset of pain
o Acute-ischemia ,meckels diverticulum,angiodysplasia
o Chronic—IBD,polyp
 Location of mass
o If near tail bone-pilondial sinus
o In or around anal opening-anal condhyloma
 Characteristic of pain
o Tearing pain- anal fissure
o Severe pain-abcess,hemorrhoid(external),fistula
o Painless-polyp, meckles diverticulum
o Pain on passing stools - haemorrhoid
 Characteristic of mass
o Hard mass –ischemic proctitis
o Multi numbered, increased in size,irregular mass-anal condyloma
o Hair growth-pilondial disease
 Associated symptoms
o Fever , urinary retention-abcess
o Bleeding- anal fissure,haemorrhoid,anal cancer,anal condyloma
o Itching – haemorrhoid,anal cancer,anal condyloma
o Spasm(lasting several hour after bowel movement)-anal fissure
o Presene of skin tag-anal fissure anal condyloma
o Abdominal pain & bloody diarrhea- IBD,infectious diarrhea, CA colon
 Characteristic of stool
o Hard stool-cause of haemorrhoid
o Thin stool-anal cancer
o Brown stool streaked with blood-anal cause
 Any underlying disease
o Infection-abcess
o HPV- most common cause of anal condylom&anorectal cancer
o Corhns disease,HIV,TB,syphilis,leukemia-chronic anal fissure
• Past medical history
 History of trauma to anal canal
 Previous history of anorectal abcess
o May develop fistula in ano
 Any history of previous history, malignancy or STD
o Anal cancer
 Previous surgery in distal abdomen region
o May cause anorectal abcess formation
 History of trauma,radition,infection(TB,actinomycosis,Chlamydia)
o May cause anorectal abcess formation
 History of cardio vascular disease
o Ischemic proctitis(rare)
o Mesenteric embolism
 Liver disease
o Variceal bleeding
• Family history
 History of malignancy
o Colorectal cancer
 Adenomatous polyposis
• Social history
 Smoking
o Risk factor for anal cancer
 Alcohol
o Increases the risk of colorectal cancer
B) What further diagnostics will you request?
• DRE: Severe anal pain is the most common presenting complaint. A palpable mass is often detected by
inspection of the perianal area or by digital rectal examination. Physical examination may reveal necrotic
skin, bullae,or crepitus. Digital rectal examination may reveal an indurated, bulging mass above the anorectal
ring.
• Anascopy : This procedure is used to examine the walls, veins and tissue of the anus. It also helpful to
examine the surrounding structures of anus and part of the rectum..
• Proctoscopy : This procedure is used examine the distal rectum and anal canal and it is also helpful to do
biopsy of the mass.
• Mostly we can confirm our diagnosis by these procedures.
• But Patients are often too tender to tolerate digital rectal examination, anoscopy, or proctoscopy. At this
situation we have to go with Barium enema X-ray.
C) How will you manage this patient?
• Anorectal pain is most often secondary to
 an anal fissure,
 perirectal abscess and/or fistula
 or a thrombosed hemorrhoid.
 Other, less common causes of anorectal pain include anal canal neoplasms, perianal
skin infection, and dermatologic conditions.
 Proctalgia fugax results from levator spasm and may present without any other anorectal
findings.
• Anal fissure: First-line therapy to minimize anal trauma includes bulk agents, stool softeners, and
warm sitz baths.
 Nitroglycerin ointment has been used locally to improve blood flow but often causes severe headaches.
 oral and topical calcium channel blockers (diltiazem and nifedipine) have also been used to heal fissures
and may have fewer side effects than topical nitrates
 Botulinum toxin (Botox) causes temporary muscle paralysis by preventing acetylcholine release from
presynaptic nerve terminals.
 Injection of botulinum toxin is used in some centers as an alternative to surgical sphincterotomy for
chronic fissure.
• Surgical therapy has traditionally been recommended for chronic fissures that have failed medical
therapy
 Anorectal abscesses: Should be treated by drainage as soon as the diagnosis is established.
 If the diagnosis is in question, an examination and drainage under anesthesia are often the most
expeditious ways both to confirm the diagnosis and to treat the problem
 Antibiotics are only indicated if there is extensive overlying cellulitis or if the patient is
immunocompromised, has diabetes mellitus, or has valvular heart disease.
 surgical treatment is dictated by the location of the internal and external openings and the course of
the fistula.
 Simple intersphincteric fistulas can often be treated by fistulotomy (opening the fistulous tract),
curettage, and healing by secondary intention
• Fistula In Ano
 surgical treatment is dictated by the location of the internal and external openings and the course
of the fistula.
 Simple intersphincteric fistulas can often be treated by fistulotomy (opening the fistulous tract),
curettage, and healing by secondary intention.
THANK YOU

More Related Content

What's hot

Gastric outlet obstruction
Gastric outlet obstructionGastric outlet obstruction
Gastric outlet obstructionikramdr01
 
Open lateral internal sphincterotomy
Open lateral internal sphincterotomyOpen lateral internal sphincterotomy
Open lateral internal sphincterotomyIndian Health Journal
 
Mesenteric ischemia
Mesenteric ischemiaMesenteric ischemia
Mesenteric ischemiakrisshk1989
 
Esophageal perforation
Esophageal perforation Esophageal perforation
Esophageal perforation Jamsheer Vt
 
Surgical Management of Chronic Pancreatitis
Surgical Management of Chronic PancreatitisSurgical Management of Chronic Pancreatitis
Surgical Management of Chronic PancreatitisHappykumar Kagathara
 
Ventral hernia by Dr Teo
Ventral hernia by Dr TeoVentral hernia by Dr Teo
Ventral hernia by Dr TeoDr. Rubz
 
Resection & anastomosis of boweL its complications PRANAYA PPT
Resection & anastomosis of boweL its complications PRANAYA PPTResection & anastomosis of boweL its complications PRANAYA PPT
Resection & anastomosis of boweL its complications PRANAYA PPTPRANAYA PANIGRAHI
 
Abdominoperineal resection.pptx
Abdominoperineal resection.pptxAbdominoperineal resection.pptx
Abdominoperineal resection.pptxmasoom parwez
 
Hydrocele- All types & treatment options
Hydrocele- All types & treatment optionsHydrocele- All types & treatment options
Hydrocele- All types & treatment optionsPawanKurliye
 
Principles of bowel anastomosis
Principles of bowel  anastomosisPrinciples of bowel  anastomosis
Principles of bowel anastomosisBashir BnYunus
 
Surgical anatomy anal canal
Surgical anatomy  anal canalSurgical anatomy  anal canal
Surgical anatomy anal canalBilal Mansoor
 
Mesenteric cyst - Journal club
Mesenteric cyst - Journal clubMesenteric cyst - Journal club
Mesenteric cyst - Journal clubPriyadarshan Konar
 

What's hot (20)

Gastric outlet obstruction
Gastric outlet obstructionGastric outlet obstruction
Gastric outlet obstruction
 
LAPAROSCOPIC APPENDECTOMY
LAPAROSCOPIC APPENDECTOMYLAPAROSCOPIC APPENDECTOMY
LAPAROSCOPIC APPENDECTOMY
 
Open lateral internal sphincterotomy
Open lateral internal sphincterotomyOpen lateral internal sphincterotomy
Open lateral internal sphincterotomy
 
Ppp pneumoperitoneum
Ppp pneumoperitoneumPpp pneumoperitoneum
Ppp pneumoperitoneum
 
Mesenteric ischemia
Mesenteric ischemiaMesenteric ischemia
Mesenteric ischemia
 
Esophageal perforation
Esophageal perforation Esophageal perforation
Esophageal perforation
 
Splenectomy
SplenectomySplenectomy
Splenectomy
 
Surgical Management of Chronic Pancreatitis
Surgical Management of Chronic PancreatitisSurgical Management of Chronic Pancreatitis
Surgical Management of Chronic Pancreatitis
 
Ventral hernia by Dr Teo
Ventral hernia by Dr TeoVentral hernia by Dr Teo
Ventral hernia by Dr Teo
 
Resection & anastomosis of boweL its complications PRANAYA PPT
Resection & anastomosis of boweL its complications PRANAYA PPTResection & anastomosis of boweL its complications PRANAYA PPT
Resection & anastomosis of boweL its complications PRANAYA PPT
 
Fistula in-ano
Fistula in-ano Fistula in-ano
Fistula in-ano
 
Anorectal diseases
Anorectal diseasesAnorectal diseases
Anorectal diseases
 
Abdominoperineal resection.pptx
Abdominoperineal resection.pptxAbdominoperineal resection.pptx
Abdominoperineal resection.pptx
 
Rif mass
Rif massRif mass
Rif mass
 
Hydrocele- All types & treatment options
Hydrocele- All types & treatment optionsHydrocele- All types & treatment options
Hydrocele- All types & treatment options
 
LOWER GI BLEEDING
LOWER GI BLEEDINGLOWER GI BLEEDING
LOWER GI BLEEDING
 
Principles of bowel anastomosis
Principles of bowel  anastomosisPrinciples of bowel  anastomosis
Principles of bowel anastomosis
 
Damage control surgery
Damage  control  surgeryDamage  control  surgery
Damage control surgery
 
Surgical anatomy anal canal
Surgical anatomy  anal canalSurgical anatomy  anal canal
Surgical anatomy anal canal
 
Mesenteric cyst - Journal club
Mesenteric cyst - Journal clubMesenteric cyst - Journal club
Mesenteric cyst - Journal club
 

Similar to Anorectal diseases

Prolapse rectum
Prolapse rectumProlapse rectum
Prolapse rectumDr KAMBLE
 
4337896.ppt
4337896.ppt4337896.ppt
4337896.pptSUBINSA2
 
Inguinal Swelling and Undescended Testis (1).pdf
Inguinal Swelling and Undescended Testis (1).pdfInguinal Swelling and Undescended Testis (1).pdf
Inguinal Swelling and Undescended Testis (1).pdfMuhammadUwais16
 
The Pharynx and Esophagus by Dr. Omar Nouri
The Pharynx and Esophagus by Dr. Omar NouriThe Pharynx and Esophagus by Dr. Omar Nouri
The Pharynx and Esophagus by Dr. Omar NouriOmar Nouri
 
Large intestine/Ulcerative colitis/colorectal carcinoma/polyp/FAP/HNPCC
Large intestine/Ulcerative colitis/colorectal carcinoma/polyp/FAP/HNPCCLarge intestine/Ulcerative colitis/colorectal carcinoma/polyp/FAP/HNPCC
Large intestine/Ulcerative colitis/colorectal carcinoma/polyp/FAP/HNPCCRajeevPandit10
 
FEMALEperineum.pptxAssignment Presentation
FEMALEperineum.pptxAssignment PresentationFEMALEperineum.pptxAssignment Presentation
FEMALEperineum.pptxAssignment PresentationAmanuelIbrahim
 
Surgical anatomy of inguinal canal
Surgical anatomy of inguinal canalSurgical anatomy of inguinal canal
Surgical anatomy of inguinal canalrks sivasankar
 
Anatomy of inguinal canal - Dr nesar Ahmad
Anatomy of inguinal canal -  Dr nesar AhmadAnatomy of inguinal canal -  Dr nesar Ahmad
Anatomy of inguinal canal - Dr nesar AhmadStudent
 
Inguinal hernia and its management
Inguinal hernia and its managementInguinal hernia and its management
Inguinal hernia and its managementJaydeep Malakar
 
Rectum & Anal canal By Prof.Dr.N.Mugunthan
Rectum & Anal canal  By Prof.Dr.N.MugunthanRectum & Anal canal  By Prof.Dr.N.Mugunthan
Rectum & Anal canal By Prof.Dr.N.MugunthanMUGUNTHAN Dr.Mugunthan
 

Similar to Anorectal diseases (20)

Rectal Prolapse
Rectal ProlapseRectal Prolapse
Rectal Prolapse
 
Prolapse rectum
Prolapse rectumProlapse rectum
Prolapse rectum
 
4337896.ppt
4337896.ppt4337896.ppt
4337896.ppt
 
Rectum And Anus
Rectum And AnusRectum And Anus
Rectum And Anus
 
Rectum And Anus
Rectum And  AnusRectum And  Anus
Rectum And Anus
 
Inguinal Swelling and Undescended Testis (1).pdf
Inguinal Swelling and Undescended Testis (1).pdfInguinal Swelling and Undescended Testis (1).pdf
Inguinal Swelling and Undescended Testis (1).pdf
 
The Pharynx and Esophagus by Dr. Omar Nouri
The Pharynx and Esophagus by Dr. Omar NouriThe Pharynx and Esophagus by Dr. Omar Nouri
The Pharynx and Esophagus by Dr. Omar Nouri
 
Fistula in ano
Fistula in anoFistula in ano
Fistula in ano
 
Rectum & Anal Canal.pptx
Rectum & Anal Canal.pptxRectum & Anal Canal.pptx
Rectum & Anal Canal.pptx
 
Large intestine/Ulcerative colitis/colorectal carcinoma/polyp/FAP/HNPCC
Large intestine/Ulcerative colitis/colorectal carcinoma/polyp/FAP/HNPCCLarge intestine/Ulcerative colitis/colorectal carcinoma/polyp/FAP/HNPCC
Large intestine/Ulcerative colitis/colorectal carcinoma/polyp/FAP/HNPCC
 
FEMALEperineum.pptxAssignment Presentation
FEMALEperineum.pptxAssignment PresentationFEMALEperineum.pptxAssignment Presentation
FEMALEperineum.pptxAssignment Presentation
 
Surgical anatomy of inguinal canal
Surgical anatomy of inguinal canalSurgical anatomy of inguinal canal
Surgical anatomy of inguinal canal
 
Oesophagoscopy
OesophagoscopyOesophagoscopy
Oesophagoscopy
 
Anatomy of oesophagus
Anatomy of oesophagusAnatomy of oesophagus
Anatomy of oesophagus
 
Anatomy of inguinal canal - Dr nesar Ahmad
Anatomy of inguinal canal -  Dr nesar AhmadAnatomy of inguinal canal -  Dr nesar Ahmad
Anatomy of inguinal canal - Dr nesar Ahmad
 
PANCREAS.pptx
PANCREAS.pptxPANCREAS.pptx
PANCREAS.pptx
 
Inguinal hernia and its management
Inguinal hernia and its managementInguinal hernia and its management
Inguinal hernia and its management
 
Urethra anatomy 2
Urethra  anatomy 2Urethra  anatomy 2
Urethra anatomy 2
 
inguinal hernia anatomy
inguinal hernia anatomy inguinal hernia anatomy
inguinal hernia anatomy
 
Rectum & Anal canal By Prof.Dr.N.Mugunthan
Rectum & Anal canal  By Prof.Dr.N.MugunthanRectum & Anal canal  By Prof.Dr.N.Mugunthan
Rectum & Anal canal By Prof.Dr.N.Mugunthan
 

Recently uploaded

Call Girl Service Bidadi - For 7001305949 Cheap & Best with original Photos
Call Girl Service Bidadi - For 7001305949 Cheap & Best with original PhotosCall Girl Service Bidadi - For 7001305949 Cheap & Best with original Photos
Call Girl Service Bidadi - For 7001305949 Cheap & Best with original Photosnarwatsonia7
 
Russian Call Girl Brookfield - 7001305949 Escorts Service 50% Off with Cash O...
Russian Call Girl Brookfield - 7001305949 Escorts Service 50% Off with Cash O...Russian Call Girl Brookfield - 7001305949 Escorts Service 50% Off with Cash O...
Russian Call Girl Brookfield - 7001305949 Escorts Service 50% Off with Cash O...narwatsonia7
 
High Profile Call Girls Jaipur Vani 8445551418 Independent Escort Service Jaipur
High Profile Call Girls Jaipur Vani 8445551418 Independent Escort Service JaipurHigh Profile Call Girls Jaipur Vani 8445551418 Independent Escort Service Jaipur
High Profile Call Girls Jaipur Vani 8445551418 Independent Escort Service Jaipurparulsinha
 
Call Girls Service Nandiambakkam | 7001305949 At Low Cost Cash Payment Booking
Call Girls Service Nandiambakkam | 7001305949 At Low Cost Cash Payment BookingCall Girls Service Nandiambakkam | 7001305949 At Low Cost Cash Payment Booking
Call Girls Service Nandiambakkam | 7001305949 At Low Cost Cash Payment BookingNehru place Escorts
 
Bangalore Call Girls Majestic 📞 9907093804 High Profile Service 100% Safe
Bangalore Call Girls Majestic 📞 9907093804 High Profile Service 100% SafeBangalore Call Girls Majestic 📞 9907093804 High Profile Service 100% Safe
Bangalore Call Girls Majestic 📞 9907093804 High Profile Service 100% Safenarwatsonia7
 
VIP Call Girls Mumbai Arpita 9910780858 Independent Escort Service Mumbai
VIP Call Girls Mumbai Arpita 9910780858 Independent Escort Service MumbaiVIP Call Girls Mumbai Arpita 9910780858 Independent Escort Service Mumbai
VIP Call Girls Mumbai Arpita 9910780858 Independent Escort Service Mumbaisonalikaur4
 
Low Rate Call Girls Pune Esha 9907093804 Short 1500 Night 6000 Best call girl...
Low Rate Call Girls Pune Esha 9907093804 Short 1500 Night 6000 Best call girl...Low Rate Call Girls Pune Esha 9907093804 Short 1500 Night 6000 Best call girl...
Low Rate Call Girls Pune Esha 9907093804 Short 1500 Night 6000 Best call girl...Miss joya
 
Call Girls ITPL Just Call 7001305949 Top Class Call Girl Service Available
Call Girls ITPL Just Call 7001305949 Top Class Call Girl Service AvailableCall Girls ITPL Just Call 7001305949 Top Class Call Girl Service Available
Call Girls ITPL Just Call 7001305949 Top Class Call Girl Service Availablenarwatsonia7
 
Ahmedabad Call Girls CG Road 🔝9907093804 Short 1500 💋 Night 6000
Ahmedabad Call Girls CG Road 🔝9907093804  Short 1500  💋 Night 6000Ahmedabad Call Girls CG Road 🔝9907093804  Short 1500  💋 Night 6000
Ahmedabad Call Girls CG Road 🔝9907093804 Short 1500 💋 Night 6000aliya bhat
 
Call Girls Frazer Town Just Call 7001305949 Top Class Call Girl Service Avail...
Call Girls Frazer Town Just Call 7001305949 Top Class Call Girl Service Avail...Call Girls Frazer Town Just Call 7001305949 Top Class Call Girl Service Avail...
Call Girls Frazer Town Just Call 7001305949 Top Class Call Girl Service Avail...narwatsonia7
 
Call Girls Jayanagar Just Call 7001305949 Top Class Call Girl Service Available
Call Girls Jayanagar Just Call 7001305949 Top Class Call Girl Service AvailableCall Girls Jayanagar Just Call 7001305949 Top Class Call Girl Service Available
Call Girls Jayanagar Just Call 7001305949 Top Class Call Girl Service Availablenarwatsonia7
 
Call Girls Kanakapura Road Just Call 7001305949 Top Class Call Girl Service A...
Call Girls Kanakapura Road Just Call 7001305949 Top Class Call Girl Service A...Call Girls Kanakapura Road Just Call 7001305949 Top Class Call Girl Service A...
Call Girls Kanakapura Road Just Call 7001305949 Top Class Call Girl Service A...narwatsonia7
 
Hemostasis Physiology and Clinical correlations by Dr Faiza.pdf
Hemostasis Physiology and Clinical correlations by Dr Faiza.pdfHemostasis Physiology and Clinical correlations by Dr Faiza.pdf
Hemostasis Physiology and Clinical correlations by Dr Faiza.pdfMedicoseAcademics
 
Housewife Call Girls Hoskote | 7001305949 At Low Cost Cash Payment Booking
Housewife Call Girls Hoskote | 7001305949 At Low Cost Cash Payment BookingHousewife Call Girls Hoskote | 7001305949 At Low Cost Cash Payment Booking
Housewife Call Girls Hoskote | 7001305949 At Low Cost Cash Payment Bookingnarwatsonia7
 
Call Girls Electronic City Just Call 7001305949 Top Class Call Girl Service A...
Call Girls Electronic City Just Call 7001305949 Top Class Call Girl Service A...Call Girls Electronic City Just Call 7001305949 Top Class Call Girl Service A...
Call Girls Electronic City Just Call 7001305949 Top Class Call Girl Service A...narwatsonia7
 
Aspirin presentation slides by Dr. Rewas Ali
Aspirin presentation slides by Dr. Rewas AliAspirin presentation slides by Dr. Rewas Ali
Aspirin presentation slides by Dr. Rewas AliRewAs ALI
 
College Call Girls Pune Mira 9907093804 Short 1500 Night 6000 Best call girls...
College Call Girls Pune Mira 9907093804 Short 1500 Night 6000 Best call girls...College Call Girls Pune Mira 9907093804 Short 1500 Night 6000 Best call girls...
College Call Girls Pune Mira 9907093804 Short 1500 Night 6000 Best call girls...Miss joya
 
Call Girls Service Chennai Jiya 7001305949 Independent Escort Service Chennai
Call Girls Service Chennai Jiya 7001305949 Independent Escort Service ChennaiCall Girls Service Chennai Jiya 7001305949 Independent Escort Service Chennai
Call Girls Service Chennai Jiya 7001305949 Independent Escort Service ChennaiNehru place Escorts
 
Russian Call Girls Chickpet - 7001305949 Booking and charges genuine rate for...
Russian Call Girls Chickpet - 7001305949 Booking and charges genuine rate for...Russian Call Girls Chickpet - 7001305949 Booking and charges genuine rate for...
Russian Call Girls Chickpet - 7001305949 Booking and charges genuine rate for...narwatsonia7
 

Recently uploaded (20)

Call Girl Service Bidadi - For 7001305949 Cheap & Best with original Photos
Call Girl Service Bidadi - For 7001305949 Cheap & Best with original PhotosCall Girl Service Bidadi - For 7001305949 Cheap & Best with original Photos
Call Girl Service Bidadi - For 7001305949 Cheap & Best with original Photos
 
Russian Call Girl Brookfield - 7001305949 Escorts Service 50% Off with Cash O...
Russian Call Girl Brookfield - 7001305949 Escorts Service 50% Off with Cash O...Russian Call Girl Brookfield - 7001305949 Escorts Service 50% Off with Cash O...
Russian Call Girl Brookfield - 7001305949 Escorts Service 50% Off with Cash O...
 
High Profile Call Girls Jaipur Vani 8445551418 Independent Escort Service Jaipur
High Profile Call Girls Jaipur Vani 8445551418 Independent Escort Service JaipurHigh Profile Call Girls Jaipur Vani 8445551418 Independent Escort Service Jaipur
High Profile Call Girls Jaipur Vani 8445551418 Independent Escort Service Jaipur
 
Call Girls Service Nandiambakkam | 7001305949 At Low Cost Cash Payment Booking
Call Girls Service Nandiambakkam | 7001305949 At Low Cost Cash Payment BookingCall Girls Service Nandiambakkam | 7001305949 At Low Cost Cash Payment Booking
Call Girls Service Nandiambakkam | 7001305949 At Low Cost Cash Payment Booking
 
Bangalore Call Girls Majestic 📞 9907093804 High Profile Service 100% Safe
Bangalore Call Girls Majestic 📞 9907093804 High Profile Service 100% SafeBangalore Call Girls Majestic 📞 9907093804 High Profile Service 100% Safe
Bangalore Call Girls Majestic 📞 9907093804 High Profile Service 100% Safe
 
VIP Call Girls Mumbai Arpita 9910780858 Independent Escort Service Mumbai
VIP Call Girls Mumbai Arpita 9910780858 Independent Escort Service MumbaiVIP Call Girls Mumbai Arpita 9910780858 Independent Escort Service Mumbai
VIP Call Girls Mumbai Arpita 9910780858 Independent Escort Service Mumbai
 
Escort Service Call Girls In Sarita Vihar,, 99530°56974 Delhi NCR
Escort Service Call Girls In Sarita Vihar,, 99530°56974 Delhi NCREscort Service Call Girls In Sarita Vihar,, 99530°56974 Delhi NCR
Escort Service Call Girls In Sarita Vihar,, 99530°56974 Delhi NCR
 
Low Rate Call Girls Pune Esha 9907093804 Short 1500 Night 6000 Best call girl...
Low Rate Call Girls Pune Esha 9907093804 Short 1500 Night 6000 Best call girl...Low Rate Call Girls Pune Esha 9907093804 Short 1500 Night 6000 Best call girl...
Low Rate Call Girls Pune Esha 9907093804 Short 1500 Night 6000 Best call girl...
 
Call Girls ITPL Just Call 7001305949 Top Class Call Girl Service Available
Call Girls ITPL Just Call 7001305949 Top Class Call Girl Service AvailableCall Girls ITPL Just Call 7001305949 Top Class Call Girl Service Available
Call Girls ITPL Just Call 7001305949 Top Class Call Girl Service Available
 
Ahmedabad Call Girls CG Road 🔝9907093804 Short 1500 💋 Night 6000
Ahmedabad Call Girls CG Road 🔝9907093804  Short 1500  💋 Night 6000Ahmedabad Call Girls CG Road 🔝9907093804  Short 1500  💋 Night 6000
Ahmedabad Call Girls CG Road 🔝9907093804 Short 1500 💋 Night 6000
 
Call Girls Frazer Town Just Call 7001305949 Top Class Call Girl Service Avail...
Call Girls Frazer Town Just Call 7001305949 Top Class Call Girl Service Avail...Call Girls Frazer Town Just Call 7001305949 Top Class Call Girl Service Avail...
Call Girls Frazer Town Just Call 7001305949 Top Class Call Girl Service Avail...
 
Call Girls Jayanagar Just Call 7001305949 Top Class Call Girl Service Available
Call Girls Jayanagar Just Call 7001305949 Top Class Call Girl Service AvailableCall Girls Jayanagar Just Call 7001305949 Top Class Call Girl Service Available
Call Girls Jayanagar Just Call 7001305949 Top Class Call Girl Service Available
 
Call Girls Kanakapura Road Just Call 7001305949 Top Class Call Girl Service A...
Call Girls Kanakapura Road Just Call 7001305949 Top Class Call Girl Service A...Call Girls Kanakapura Road Just Call 7001305949 Top Class Call Girl Service A...
Call Girls Kanakapura Road Just Call 7001305949 Top Class Call Girl Service A...
 
Hemostasis Physiology and Clinical correlations by Dr Faiza.pdf
Hemostasis Physiology and Clinical correlations by Dr Faiza.pdfHemostasis Physiology and Clinical correlations by Dr Faiza.pdf
Hemostasis Physiology and Clinical correlations by Dr Faiza.pdf
 
Housewife Call Girls Hoskote | 7001305949 At Low Cost Cash Payment Booking
Housewife Call Girls Hoskote | 7001305949 At Low Cost Cash Payment BookingHousewife Call Girls Hoskote | 7001305949 At Low Cost Cash Payment Booking
Housewife Call Girls Hoskote | 7001305949 At Low Cost Cash Payment Booking
 
Call Girls Electronic City Just Call 7001305949 Top Class Call Girl Service A...
Call Girls Electronic City Just Call 7001305949 Top Class Call Girl Service A...Call Girls Electronic City Just Call 7001305949 Top Class Call Girl Service A...
Call Girls Electronic City Just Call 7001305949 Top Class Call Girl Service A...
 
Aspirin presentation slides by Dr. Rewas Ali
Aspirin presentation slides by Dr. Rewas AliAspirin presentation slides by Dr. Rewas Ali
Aspirin presentation slides by Dr. Rewas Ali
 
College Call Girls Pune Mira 9907093804 Short 1500 Night 6000 Best call girls...
College Call Girls Pune Mira 9907093804 Short 1500 Night 6000 Best call girls...College Call Girls Pune Mira 9907093804 Short 1500 Night 6000 Best call girls...
College Call Girls Pune Mira 9907093804 Short 1500 Night 6000 Best call girls...
 
Call Girls Service Chennai Jiya 7001305949 Independent Escort Service Chennai
Call Girls Service Chennai Jiya 7001305949 Independent Escort Service ChennaiCall Girls Service Chennai Jiya 7001305949 Independent Escort Service Chennai
Call Girls Service Chennai Jiya 7001305949 Independent Escort Service Chennai
 
Russian Call Girls Chickpet - 7001305949 Booking and charges genuine rate for...
Russian Call Girls Chickpet - 7001305949 Booking and charges genuine rate for...Russian Call Girls Chickpet - 7001305949 Booking and charges genuine rate for...
Russian Call Girls Chickpet - 7001305949 Booking and charges genuine rate for...
 

Anorectal diseases

  • 1. ANORECTAL DISEASES SATHIYAMURTHY LOGESHWARRAJ 17-0691-378 BLOCK 10 GROUP 5 CLERKSHIP INTERNATIONAL MARCH BATCH 2021 SURGERY ROTATION DEPARTMENT OF SURGERY JONELTA FOUNDATION SCHOOL OF MEDICINE UNIVERSITY OF PEPETUAL HELP RIZAL
  • 2. 1. Discuss the basic anatomy of the anal canal and rectum
  • 3. Anatomy of the Anal Canal and Rectum Landmarks ● The rectum is approximately 12 to 15 cm in length ● Valves of Houston ○ Three distinct submucosal folds that extend into the rectal lumen. ● Posteriorly, the presecral fascia separates the rectum from the presacral venous plexus and the pelvic nerves.
  • 4. Anatomy of the Anal Canal and Rectum ● At S4, the rectosacral fascia (Waldeyer’s fascia) extends anteriorly and caudally and attaches to the fascia propria at the anorectal junction. ● Anteriorly, Denonvilliers’ fascia separates the rectum from the prostate and seminal vesicles in men and from the vagina in women. ● The lateral ligaments support the lower rectum
  • 5. Anatomy of the Anal Canal and Rectum The Anatomic Anal Canal ● Extends from the dentate or pectinate line to the anal verge ● The dentate or pectinate line marks the transition point between columnar rectal mucosa and squamous anoderm. ● The anal transition zone ○ includes mucosa proximal to the dentate line that shares histologic characteristics of columnar, cuboidal, and squamous epithelium. ○ Extends only 1 to 2 cm proximal to the dentate line, however this value is highly variable and can be as far as 15 cm proximal to the dentate line.
  • 6. Anatomy of the Anal Canal and Rectum The Anatomic Anal Canal ● The dentate line is surrounded by longitudinal mucosal folds, known as the columns of Morgagni, into which the anal crypts empty ○ These crypts are the source of cryptoglandular abscesses
  • 7. Anatomy of the Anal Canal and Rectum The Surgical Anal Canal ● Begins at the anorectal junction and terminates at the anal verge. ● Measures 2 to 4 cm in length and is generally longer in men than in women.
  • 8.
  • 9. Anatomy of the Anal Canal and Rectum Distal Rectum ● The inner smooth muscle is thickened and comprises the internal anal sphincter that is surrounded by the subcutaneous, superficial, and deep external sphincter. ● The deep external anal sphincter is an extension of the puborectalis muscle. ● The puborectalis, iliococcygeus, and pubococcygeus muscles form the levator ani muscle of the pelvic floor
  • 10. Anorectal Arterial Supply ● The superior rectal artery ○ arises from the terminal branch of the inferior mesenteric artery ○ supplies the upper rectum. ● The middle rectal artery ○ arises from the internal iliac ○ the presence and size of these arteries are highly variable.
  • 11. Anorectal Arterial Supply ● The inferior rectal artery ○ arises from the internal pudendal artery (a branch of the internal iliac artery) ● A rich network of collaterals connects the terminal arterioles of each of these arteries, thus making the rectum relatively resistant to ischemia
  • 12. Anorectal Venous Drainage ● The venous drainage of the rectum parallels the arterial supply. ● The superior rectal vein drains into the portal system via the inferior mesenteric vein. ● The middle rectal vein drains into the internal iliac vein.
  • 13. Anorectal Venous Drainage ● The inferior rectal vein drains into the internal pudendal vein, and subsequently into the internal iliac vein. ● A submucosal plexus deep to the columns of Morgagni forms the hemorrhoidal plexus and drains into all three veins.
  • 14. Lymphatic Drainage Rectal ● Parallels the vascular supply. ● Lymphatic channels in the upper and middle rectum drain superiorly into the inferior mesenteric lymph nodes. ● Lymphatic channels in the lower rectum drain both superiorly into the inferior mesenteric lymph nodes and laterally into the internal iliac lymph nodes.
  • 15. Lymphatic Drainage Anal Canal ● More complex pattern of lymphatic drainage. ● Proximal to the dentate line, lymph drains into both the inferior mesenteric lymph nodes and the internal iliac lymph nodes. ● Distal to the dentate line, lymph primarily drains into the inguinal lymph nodes, but can also drain into the inferior mesenteric lymph nodes and internal iliac lymph nodes.
  • 16.
  • 17. Anorectal Nerve Supply ● Both sympathetic and parasympathetic nerves innervate the anorectum. ○ Sympathetic nerve fibers are derived from L1–L3 and join the preaortic plexus. ■ The preaortic nerve fibers then extend below the aorta to form the hypogastric plexus, which subsequently joins the parasympathetic fibers to form the pelvic plexus.
  • 18. Anorectal Nerve Supply ○ Parasympathetic nerve fibers are known as the nervi erigentes and originate from S2–S4 ■ These fibers join the sympathetic fibers to form the pelvic plexus. ● Sympathetic and parasympathetic fibers then supply the anorectum and adjacent urogenital organs.
  • 19. Anorectal Nerve Supply ● The internal anal sphincter ○ innervated by sympathetic and parasympathetic nerve fibers ■ both types of fibers inhibit sphincter contraction ● The external anal sphincter and puborectalis muscles ○ innervated by the inferior rectal branch of the internal pudendal nerve. ● The levator ani ○ Innervated by both the internal pudendal nerve and direct branches of S3 to S5.
  • 20. Anorectal Nerve Supply ● Sensory innervation to the anal canal ○ provided by the inferior rectal branch of the pudendal nerve. ○ While the rectum is relatively insensate, the anal canal below the dentate line receives somatic innervation
  • 22. 2. Discuss the basics of the mechanism of defecation
  • 23. Motility ● Unlike the small intestine, the large intestine does not demonstrate cyclic motor activity characteristic of the migratory motor complex. ● The colon displays intermittent contractions of either low or high amplitude. ● Low-amplitude, short-duration contractions occur in bursts and appear to move the colonic contents both antegrade and retrograde. ● It is thought that these bursts of motor activity delay colonic transit and thus increase the time available for absorption of water and exchange of electrolytes.
  • 24. Motility ● High-amplitude, prolonged duration, propagated contractions (HAPCs) ● Occur in a more coordinated fashion and create “mass movements,” four to ten times per day, mostly after meals and awakening. ● Bursts of “rectal motor complexes” also have been described. ● In general, cholinergic activation increases colonic motility.
  • 25. Defecation ● Defecation is a complex, coordinated mechanism involving colonic mass movement, increased intra-abdominal and rectal pressure, and relaxation of the pelvic floor. ● Distention of the rectum causes a reflex relaxation of the internal anal sphincter (the rectoanal inhibitory reflex) that allows the contents to make contact with the anal canal. ● This “sampling reflex” allows the sensory epithelium to distinguish solid stool from liquid stool and gas. ● If defecation does not occur, the rectum relaxes and the urge to defecate passes (accommodation response). ● Defecation proceeds by coordinating increasing intra-abdominal pressure via a Valsalva maneuver with rectal contraction, relaxation of the puborectalis muscle, and opening of the anal canal.
  • 26. Continence ● Continence requires: ○ Adequate rectal wall compliance to accommodate the fecal bolus ○ Appropriate neurogenic control of the pelvic floor and sphincter mechanism ○ Functional internal and external sphincter muscles ● At rest, the puborectalis muscle creates a “sling” around the distal rectum, forming a relatively acute angle that distributes intra-abdominal forces onto the pelvic floor. ○ With defecation, this angle straightens, allowing downward force to be applied along the axis of the rectum and anal canal. ● The internal and external sphincters are tonically active at rest. ○ The internal sphincter is responsible for most of the resting, involuntary sphincter tone (resting pressure). ○ The external sphincter is responsible for most of the voluntary sphincter tone (squeeze pressure).
  • 27. Continence ● Branches of the pudendal nerve innervate both the internal and external sphincter. ● The hemorrhoidal cushions may contribute to continence by mechanically blocking the anal canal. ● Finally, liquid stools exacerbate abnormalities with these anatomic and physiologic mechanisms, so a formed stool contributes to maintaining continence. ○ Thus, impaired continence may result from: ■ poor rectal compliance ■ injury to the internal ■ and/or external sphincter or puborectalis, or neuropathy
  • 28. 3. Describe and perform proper evaluation of anorectal diseases.
  • 29. Complete History and Physical Examination · Starting points for evaluation of any patient complaining with symptoms related to diseases of the colon, rectum and anus · Past medical history · Surgical History to detect underlying conditions o Resultant gastrointestinal anatomy o Anorectal surgery – abdominal or anorectal complaints · Obstetric history – occult pelvic floor and/or anal sphincter damage · Family history – colorectal disease, especially inflammatory bowel disease, polyps, and colorectal cancer o History of other malignancies – genetic syndrome · Medications – some may cause gastrointestinal symptoms o Adequacy of medical treatment · Visual inspection of anus and perineum o Digital rectal examination
  • 30. Endoscopy Anoscopy · Instrument used to examine the anal canal · Measures approximately 8 cm in length · Larger anoscope – anal procedures o Rubber band ligation o Inserted into the anal canal o Obturator is withdrawn, inspection is done and the anoscope withdrawn o Rotated 90 degrees and reinserted to allow visualization of all four quadrants of the canal o If patient can’t tolerate DRE, anoscopy should not be attempted
  • 31. Proctoscopy · Rigid proctoscope – examination of rectum and distal sigmoid colon · Ocassionally used to therapeutics · 25 cm in length and in various diameters (15 or 19mm) · Pediatric (11 mm) · Suction is necessary for an adequate examination · Transanal Endoscopic Microsurgery (TEM) o Much wider diameter o Can be used for excision of large polyps and tumors · Transanal Minimally Invasice Surgery (TAMIS) o Can achieve similar resections to TEM but does not utilize a proctoscope and depends on insufflation
  • 32. Flexible Sigmoidoscopy and Colonoscopy · Video or fiberoptic sigmoidoscopy and colonoscopy · Sigmoidoscopes measures 60cm in length · Full depth may allow visualization until the splenic flexure · Partial preparation with enemas is adequate · Most patients can tolerate this without sedation · Colonoscopes measure 100-160cm in length can examine the the entire colon and terminal ileum · Complete bowel preparation is usually necessary · Duration and discomfort usually requires conscious sedation · Electrocautery should not be used in the absence of complete bowel preparation o Risk for explosion of intestinal methane or hydrogen gases · Colonoscopes –single channel where snares, biopsy forceps or electrocautery can be passed o Suction and irrigation capability o Therapeutic colonoscopies possess two channels – simultaneous suction/irrigation and use of snares, biopsy forceps and electrocautery
  • 33. Capsule Endoscopy · Small ingestible camera · Images of mucosa and GIT are captured · Transmitted by radiofrequency to a belt-held receiver and to the computer · Used to detect small bowel lesions · Possibility of an acute obstruction led to: o Dissolvable capsule
  • 34. Imaging Plain X-rays and Contrast Studies · Plain x-rays (supine, upright and diaphragmatic views) o Detects free intra-abdominal air o Bowel gas patterns – obstruction o Volvulus · Obstructive symptoms · Delineating fistulous tracts · Diagnosing small perforations · Anastomotic leaks · Gastrografin cannot provide mucosal detail provided by barium, this water-soluble agent is recommended for perforation or leak · Double-contrast barium enema (followed by insufflation of air) o 70-90% sensitive mass lesions greater than 1cm in diameter
  • 35. Computed Tomography · Detection of extraluminal disease o Intra-abdominal abscesses o Pericolic inflammation o Staging colorectal carcinoma  Sensitivity in detecting hepatic metastases o Extravasation of oral or rectal contrast  Perforation or anatomic leak o Nonspecific findings  Bowel wall thickening  Mesenteric stranding · Inflammatory bowel disease · Enteritis/colitis · Ischemia o Standard CT Scan  Insensitive for the detection of intraluminal lesions
  • 36. Computed Tomography Colonography · Virtual Colonoscopy · Designed to overcome the limitations of conventional CT scanning · Helical CT and 3D reconstruction to detect intraluminal colonic lesions · Oral bowel preparation, oral and rectal contrast, colon insufflation may maximize sensitivity · 85-90% sensitivity and specificity in detecting 1cm or larger polyps · Alternative for traditional colonoscopy
  • 37. Magnetic Resonance Imaging · Evaluation of pelvic lesions · More sensitive than CT in detecting bony involvement or pelvic sidewall extention of rectal tumors · Accurate in detecting extent of rectal cancer spread to adjacent mesorectum · Reliably predict difficulty in achieving radial margin clearance of a rectal cance · If radial margin is threatened, neoadjuvant chemoradiation is indicated · Helpful in detection and delineation of complex fistulas in ano · Endorectal coil increases sensitivity
  • 38. Positron Emission Tomography · Imaging tissues with high levels of anaerobic glycolysis – malignant tumors · F-fluorode-oxyglucose (FDG) is injected as a tracer o Metabolism of this molecule results in positron emission · Adjunct to CT scan in the staging of colorectal cancer · Useful in discriminating recurrent cancer from fibrosis · PET/CT o Anatomic regions of high isotope accumulation (hot spots on PET) o Increasingly used to diagnose recurrent and/or metastatic colorectal cancer
  • 39. Scintigraphy to Assess Gastrointestinal Bleeding · Technetium 99-tagged RBC scan · Nuclear medicine that tests Tc-erythrocytes and dynamic images to localize a bleeding source · Patients actively bleeding at the time of imaging · Normal distribution of Tc-erythrocytes in vasculature, liver, spleen, penile circulation with mild uptake in kidneys and bladder o Can interfere with localization in bowel segments near those given structures · Patients must be stable to tolerate imaging up to 4 hours with slow bleeding rate of 0.05-2.0mL/min
  • 40. Single Photon Emission Computed Tomography (SPECT/CT) · Radiolabeled RBCs are used · Cross-sectional images provide a more specific location of the bleeding source
  • 41. Angiography · Ocassionally used for detection of bleeding within the colon or small bowel · Visualize hemorrhage o Brisk bleeding (0.5-1.0mL/min) · If extravasation of contrast identified, infusion of vasopression or angiographic embolixation can be therapeutic · If surgical resection required, angiographic catheter can be left in place to assist in the identification of bleeding site intraoperatively · CT and MRI angiography are also useful in assessing patency of visceral vessels · Technique uses 3D reconstruction to detect vascular lesions · If there is an abnormality, traditional techniques may be used for further definition of the problem
  • 42. Endorectal and Endoanal Ultrasound · Primarily used in the evaluation of the depth of invasion of neoplastic lesions in the rectum · Normal rectum wall appears as a five-layer structure such as this one · Can reliably differentiate most benign polyp from invasive tumors based on the integrity of the submucosal layer · Can also differentiate superficial T1-T2 from deeper T3- T4 tumors · Useful in the evaluation of patients with incontinence, constipation, rectal prolapse, obstructed defection and other functional disorders of the pelvic floor
  • 43. Manometry · Placing a pressure-sensitive catheter in the lower rectum · Catheter withdrawn through the anal canal and pressures recorded · Balloon is attached to the tip of the catheter · The resting pressure in the anal canal reflects the function of the internal anal sphincter (N= 40-80mmHg above resting pressure) · High pressure zone estimates lengh of the anal canal (N=2.0-4.0cm) · The rectoanal inhibitory reflex can be detected by inflating a balloon in the distal rectum o Absence of this reflex = Hirschsprung Disease
  • 44. Neurophysiology · Fuction of the pudendal nerves and recruitment of puborectalis muscle fibers · Pudenda; nerve terminal motor latency measures peed of transmission of a nerve impulse through distal pudendal nerve fibers (N=1.8-2.2ms) o Prolonged latency = presence of neuropathy · EMG recruitment assesses the contraction and relaxation of the puborectalis muscels during attempted defacation o Recruitment increases when a patient is instructed to squeeze o Decreases when instructed to push o Inappropriate recruitment is an indication of paradoxical contraction · Needle EMG has been used to map both the pudendal nerves and the anatomy of the internal and external sphincters · Painful and poorly tolerated by most patients
  • 45. Rectal Evacuation Studies · Balloon expulsion test and video defecography · Balloon expulsion - Patient’s ability to expel and intrarectal balloon · Video defecography – more detailed assessment of defecation o Barium paste is placed in the rectum and defecation is recorded fluoroscopically o Help diagnose - Obstructed defecation from nonrelaxation of the puborectalis muscle or anal sphincter dyssynergy - Increased perineal descent - Rectal prolapse - Intussusception - Rectocele - Enterocele o Addition of vaginal contrast and intraperitoneal contrast
  • 46. 4. Discuss the characteristic history findings for benign and malignant anal pathology
  • 47. Pain ● Abdominal Pain ○ Is a nonspecific symptom with myriad causes. ○ Abdominal pain related to the colon and rectum can result from: ■ Obstruction (either inflammatory or neoplastic) ■ inflammation ■ Perforation ■ Ischemia. ○ Plain X-rays and judicious use of contrast studies and/or a CT scan can often confirm the diagnosis. ○ Gentle retrograde contrast studies (Gastrografin enema) may be useful in delineating the degree of colonic obstruction. ○ Sigmoidoscopy and/or colonoscopy can assist in the diagnosis of ischemic colitis, infectious colitis, and inflammatory bowel disease. ■ However, if perforation or near complete obstruction is suspected, colonoscopy and/or sigmoidoscopy are generally contraindicated. ○ Evaluation and treatment of abdominal pain from a colorectal source should follow the usual surgical principles of a thorough history and physical examination, appropriate diagnostic tests, resuscitation, and appropriately timed surgical intervention.
  • 48. ● Pelvic Pain ○ Can originate from the distal colon and rectum or from adjacent urogenital structures. ○ Tenesmus (is a cramping rectal pain that makes you feel that you need to have a bowel movement, even if you already had one) may result from proctitis or from a rectal or retrorectal mass, or fecal impaction in a constipated patient. ○ Cyclical pain associated with menses, especially when accompanied by rectal bleeding, suggests a diagnosis of endometriosis. ○ Pelvic inflammatory disease also can produce significant abdominal and pelvic pain. ○ The extension of a peridiverticular abscess or periappendiceal abscess into the pelvis may also cause pain. ○ CT scan and/or MRI may be useful in differentiating these diseases. ○ Proctoscopy (if tolerated) also can be helpful. ○ Occasionally, laparoscopy will yield a diagnosis, although with access to high- quality imaging, indications for diagnostic surgery should be rare.
  • 49. Lower Gastric Bleeding ● The most common source of gastrointestinal hemorrhage is esophageal, gastric, or duodenal, nasogastric aspiration should always be performed; ● Return of bile suggests that the source of bleeding is distal to the ligament of Treitz. ● Colonoscopy may identify the cause of the bleeding, and cautery or injection/application of epinephrine into the bleeding site may be used to control hemorrhage. ● A SPECT/CT may be helpful if other modalities have not achieved localization, particularly in distinguishing between small intestinal and colon sources. ● Colectomy may be required if bleeding persists despite interventions. ● Intraoperative colonoscopy and/or enteroscopy may assist in localizing bleeding.
  • 50. Lower Gastric Bleeding ● Colon neoplasms bleed intermittently and rarely present with rapid hemorrhage, the presence of occult fecal blood should always prompt a colonoscopy. ● Unexplained iron deficiency anemia is also an indication for colonoscopy. ● Hematochezia is commonly caused by hemorrhoids or a fissure. ● Sharp, knife-like pain and bright red rectal bleeding with bowel movements suggest the diagnosis of fissure. ● Painless, bright red rectal bleeding with bowel movements is often secondary to a friable internal hemorrhoid that is easily detected by anoscopy
  • 51.
  • 52. Constipation and Obstructive Defecation ● Constipation is an extremely common complaint, affecting more than 4 million people in the United States. ● Patients may describe infrequent bowel movements, hard stools, or excessive straining. ● A careful history of these symptoms often clarifies the nature of the problem. ● Constipation has many causes ○ Underlying metabolic, pharmacologic, endocrine, psychological, and neurologic causes often contribute to the problem. ● A stricture or mass lesion should be excluded by colonoscopy, barium enema, or CT colonography. ● After these causes have been excluded, evaluation focuses on differentiating slow- transit constipation from outlet obstruction.
  • 53. Constipation and Obstructive Defecation ● Transit studies, in which radiopaque markers are swallowed and then followed radiographically, are useful for diagnosing slow-transit constipation. ● In this study, patients ingest radiopaque studies and are followed radiographically for 5 days. ○ Retention of 20% or greater of these markers in the colon demonstrated slow transit. ○ If these markers are congregated in the rectosigmoid colon and rectum, obstructed defecation/outlet obstruction is suggested. ● Anorectal manometry and EMG can detect nonrelaxation of the puborectalis, which contributes to outlet obstruction. ● The absence of an anorectal inhibitory reflex suggests Hirschsprung’s disease and may prompt a rectal mucosal biopsy. ● Defecography can identify rectal prolapse, intussusception, rectocele, or enterocele.
  • 54. Constipation and Obstructive Defecation ● Medical management is the mainstay of therapy for constipation and includes fiber, increased fluid intake, and laxatives. ● Outlet obstruction from nonrelaxation of the puborectalis or anal sphincter dyssynergy often responds to biofeedback. ● Surgery to correct rectocele and rectal prolapse (with or without sigmoid resection) has a variable effect on symptoms of constipation but can be successful in selected patients. ● Subtotal colectomy is considered only for patients with severe slow-transit constipation (colonic inertia) refractory to maximal medical interventions. ● While this operation almost always increases bowel movement frequency, complaints of diarrhea, incontinence, and abdominal pain are not infrequent.
  • 55. Diarrhea and Irritable Bowel Syndrome ● Diarrhea is also a common complaint and is usually a self-limited symptom of infectious gastroenteritis. ● If diarrhea is chronic or is accompanied by bleeding or abdominal pain, further investigation is warranted. ● Bloody diarrhea and pain are characteristic of colitis; etiology can be an infection (invasive E coli, Shigella, Salmonella, Campylobacter, Entamoeba histolytica, or C difficile), inflammatory bowel disease (ulcerative colitis or Crohn’s colitis), or ischemia. ● Stool wet-mount and culture can often diagnose infection. ● Sigmoidoscopy or colonoscopy can be helpful in diagnosing inflammatory bowel disease or ischemia. ○ However, if the patient has abdominal tenderness, particularly with peritoneal signs, or any other evidence of perforation, endoscopy is contraindicated.
  • 56. Diarrhea and Irritable Bowel Syndrome ● Chronic diarrhea may present a more difficult diagnostic dilemma. ● Chronic ulcerative colitis, Crohn’s colitis, infection, malabsorption, and short gut syndrome can cause chronic diarrhea. ● Rarely, carcinoid syndrome and islet cell tumors (vasoactive intestinal peptide–secreting tumor [VIPoma], somatostatinoma, gastrinoma) present with this symptom. ● Large villous lesions may cause secretory diarrhea. ● Collagenous colitis can cause diarrhea without any obvious mucosal abnormality. ● Along with stool cultures, tests for malabsorption, and metabolic investigations, colonoscopy can be invaluable in differentiating these causes.
  • 57. Diarrhea and Irritable Bowel Syndrome ● Biopsies should be taken even if the colonic mucosa appears grossly normal. ● Irritable bowel syndrome is a particularly troubling constellation of symptoms consisting of crampy abdominal pain, bloating, constipation, and urgent diarrhea. ○ Workup reveals no underlying anatomic or physiologic abnormality. ○ Once other disorders have been excluded, dietary restrictions and avoidance of caffeine, alcohol, and tobacco may help to alleviate symptoms. ● Antispasmodics and bulking agents may be helpful
  • 58. Incontenence ● Estimated to occur in 10 to 13 individuals per 1000 people older than age 65 years. ● Incontinence ranges in severity from occasional leakage of gas and/or liquid stool to daily loss of solid stool. ● The underlying cause of incontinence is often multifactorial, and diarrhea is often contributory. ● In general, causes of incontinence can be classified as neurogenic or anatomic. ○ Neurogenic causes include diseases of the central nervous system and spinal cord along with pudendal nerve injury. ○ Anatomic causes include congenital abnormalities, procidentia (rectal prolapse), overflow incontinence secondary to impaction or an obstructing neoplasm, and trauma. ● The most common traumatic cause of incontinence is injury to the anal sphincter during vaginal delivery. ● Other causes include anorectal surgery, impalement, and pelvic fracture.
  • 59. Incontenence ● After a thorough medical evaluation to detect underlying conditions that might contribute to incontinence, evaluation focuses on assessment of the anal sphincter and pudendal nerves. ● Pudendal nerve terminal motor latency testing may detect neuropathy. ● Anal manometry can detect low resting and squeeze pressures. ● Physical examination and defecography can detect rectal prolapse. ● Endoanal ultrasound is invaluable in diagnosing sphincter defects. ● Diarrhea should be treated medically (fiber, antidiarrheal agents). ● Even in the absence of frank diarrhea, the addition of dietary fiber may improve continence.
  • 60. Incontenence ● Many patients with a sphincter defect are candidates for an overlapping sphincteroplasty. ● Sacral nerve stimulation has shown to decrease episodes of fecal incontinence and has proven durability in the long term (5 years). ● The artificial bowel sphincter may be useful in patients who fail other interventions. ● Other options include radiofrequency energy to the anal canal (SECCA procedure), magnetic anal sphincter, and injectable submucosal bulking agents, but longterm efficacy has not yet been proven. ● Finally, a stoma can provide relief for severely incontinent patients who have failed or are not candidates for other interventions.
  • 61. 5. Describe indications for inpatient and outpatient endoscopies (anoscopy vs proctoscopy vs flexible sigmoidoscopy and colonoscopy)
  • 62. Anoscopy ● Useful instrument for examination of the anal canal. ● A larger anoscope provides better exposure for anal procedures such as rubber band ligation or sclerotherapy of hemorrhoids.
  • 63. Protoscope ● Useful for examination of the rectum and distal sigmoid colon and is occasionally used therapeutically. ● Standard proctoscope is 25 cm in length and available in various diameters. ● 15- or 19-mm diameter proctoscope - used for diagnostic examinations. ● “Pediatric” proctoscope (11-mm diameter) - better tolerated by patients with anal stricture.
  • 64. Protoscope ● Transanal Endoscopic Microsurgery (TEM) ○ Has a much wider diameter ○ Can be used for excisions of large polyps and tumors. ● Transanal minimally invasive surgery (TAMIS) ○ Can achieve similar resections to TEM ○ Does not utilize a proctoscope ○ Depends on insufflation to create a working space in the rectum while utilizing a circular wound protector to open the anus.
  • 65. Flexible Sigmoidoscopy and Colonoscopy ● Provide excellent visualization of the colon and rectum. ● Sigmoidoscopes ○ Measure 60 cm in length. ○ Full depth of insertion may allow visualization as high as the splenic flexure ○ Mobility and redundancy of the sigmoid colon often limit the extent of the examination. ○ Partial preparation with enemas is usually adequate ○ Most patients can tolerate this procedure without sedation
  • 66. Flexible Sigmoidoscopy and Colonoscopy ● Colonoscopes ○ Measure 100 to 160 cm in length ○ Capable of examining the entire colon and terminal ileum. ○ A complete oral bowel preparation is usually necessary ○ Duration and discomfort of the procedure usually require conscious sedation. ○ Diagnostic colonoscopes ■ Possess a single channel through which instruments such as snares, biopsy forceps, or electrocautery can be passed; ■ This channel also provides suction and irrigation capability. ○ Therapeutic colonoscopes ■ Possess two channels to allow simultaneous suction/irrigation and the use of snares, biopsy forceps, or electrocautery.
  • 67. Flexible Sigmoidoscopy and Colonoscopy ● Both sigmoidoscopy and colonoscopy can be used diagnostically and therapeutically. ● Electrocautery should generally not be used in the absence of a complete bowel preparation because of the risk of explosion of intestinal methane or hydrogen gases.
  • 68. 6. Discuss the relevant anatomy, diagnosis and management of a patient with External and Internal hemorrhoids Anal Fissure Perirectal Abscess Proctitis Anal condyloma Pilonidal disease Fistula-in-ano Anorectal CA
  • 69. External and Internal hemorrhoids ● Hemorrhoids are cushions of submucosal tissue containing venules, arterioles, and smooth muscle fibers that are located in the anal canal ● Three hemorrhoidal cushions are found in the left lateral, right anterior, and right poste- rior positions. Hemorrhoids are thought to function as part of the continence mechanism and aid in complete closure of the anal canal at rest. ● Because hemorrhoids are a normal part of anorectal anatomy, treatment is only indicated if they become symptomatic. ● Excessive straining, increased abdominal pressure, and hard stools increase venous engorgement of the hemorrhoidal plexus and cause prolapse of hemorrhoidal tissue. ● Bleeding, thrombosis, and symptomatic hemorrhoidal prolapse may result.
  • 70. External hemorrhoids ● External hemorrhoids are located distal to the dentate line and are covered with anoderm. ○ Because the anoderm is richly innervated, thrombosis of an external hemorrhoid may cause significant pain. ○ It is for this reason that external hemorrhoids should not be ligated or excised without adequate local anesthetic. ● A skin tag is redundant fibrotic skin at the anal verge, often persisting as the residua of a thrombosed external hemorrhoid. ● Skin tags are often confused with symptomatic hemorrhoids. ● External hemorrhoids and skin tags may cause itching and difficulty with hygiene if they are large. ● Treatment of external hemorrhoids and skin tags is only indicated for symptomatic relief.
  • 71. Internal Hemorrhoids ● Internal hemorrhoids are located proximal to the dentate line and covered by insensate anorectal mucosa. Internal hemorrhoids may prolapse or bleed, but they rarely become painful unless they develop thrombosis and necrosis (usually related to severe prolapse, incarceration, and/or strangulation). ● Internal hemorrhoids are graded according to the extent of prolapse. ○ First-degree hemorrhoids bulge into the anal canal and may prolapse beyond the dentate line on straining. ○ Second-degree hemorrhoids prolapse through the anus but reduce spontaneously. ○ Third-degree hemorrhoids prolapse through the anal canal and require manual reduction. ○ Fourth-degree hemorrhoids prolapse but cannot be reduced and are at risk for strangulation.
  • 72. External and Internal Hemorrhoids ● Combined internal and external hemorrhoids straddle the dentate line and have characteristics of both internal and external hemorrhoids. ● Hemorrhoidectomy is often required for large, symptomatic, combined hemorrhoids. ● Postpartum hemorrhoids result from straining during labor, which results in edema, thrombosis, and/or strangulation. ● Hemorrhoidectomy is often the treatment of choice, especially if the patient has had chronic hemorrhoidal symptoms.
  • 73. Risk for bleeding ● Portal hypertension was long thought to increase the risk of hemorrhoidal bleeding because of the anastomoses between the portal venous system (middle and upper hemorrhoidal plexuses) and the systemic venous system (inferior rectal plexuses). ○ It is now understood that hemorrhoidal disease is no more common in patients with portal hypertension than in the normal population. ● Rectal varices, however, may occur and may cause hemorrhage in these patients. ○ In general, rectal varices are best treated by lowering portal venous pressure. ○ Rarely, suture ligation may be necessary if massive bleeding persists. ○ Surgical hemorrhoidectomy should be avoided in these patients because of the risk of massive, difficult-to-control variceal bleeding.
  • 74. Treatment ● Medical therapy ○ Bleeding from first and second-degree hemorrhoids often improves with the addition of dietary fiber, stool softeners, increased fluid intake, and avoidance of straining. ○ Associated pruritus often may improve with improved hygiene. ○ Many over-the-counter topical medications are desiccants and are relatively ineffective for treating hemorrhoidal symptoms. ● Rubber Band Ligation ○ Mucosa located 1 to 2 cm proximal to the dentate line is grasped and pulled into a rubber band applier. After firing the ligator, the rubber band strangulates the underlying tissue, causing scarring and preventing further bleeding or prolapse
  • 75. Treatment ● Infrared photocoagulation ○ Infrared photocoagulation is an effective office treatment for small first- and second-degree hemorrhoids. The instrument is applied to the apex of each hem- orrhoid to coagulate the underlying plexus. All three quadrants may be treated during the same visit. Larger hemorrhoids and hemorrhoids with a significant amount of prolapse are not effectively treated with this technique. ● Sclerotherapy ○ The injection of bleeding internal hemorrhoids with sclerosing agents is another effective office technique for treatment of first-, second-, and some third- degree hemorrhoids. One to 3 mL of a sclerosing solution (phenol in olive oil, sodium morrhuate, or quinine urea) is injected into the submucosa of each hemorrhoid. Few complications are associated with sclerotherapy, but infection and fibrosis have been reported.
  • 76. Treatment ● Excision of Thrombosed External Hemorrhoids ○ Acutely thrombosed external hemorrhoids generally cause intense pain and a palpable perianal mass during the first 24 to 72 hours after thrombosis. ○ The thrombosis can be effectively treated with an elliptical excision performed in the office under local anesthesia. ○ Because the clot is usually loculated, simple incision and drainage is rarely effective. After 72 hours, the clot begins to resorb, and the pain resolves spontaneously. ○ Excision is unnecessary, but sitz baths and analgesics are often helpful. ● Operative Hemorrhoidectomy ○ A number of surgical procedures have been described for elective resection of symptomatic hemorrhoids. ○ All are based on decreasing blood flow to the hemorrhoidal plexuses and excising redundant anoderm and mucosa.
  • 77. Treatment ● Closed Submucosal Hemorrhoidectomy ○ The Parks or Fer-guson hemorrhoidectomy involves resection of hemorrhoidal tissue and closure of the wounds with absorbable suture. ○ The procedure may be performed in the prone or lithotomy position under local, regional, or general anesthesia. ○ The anal canal is examined and an anal speculum inserted. ○ The hemorrhoid cushions and associated redundant mucosa are identified and excised using an elliptical incision starting just distal to the anal verge and extending proximally to the anorectal ring. ○ It is crucial to identify the fibers of the internal sphincter and carefully brush these away from the dissection in order to avoid injury to the sphincter. ○ The apex of the hemorrhoidal plexus is then ligated and the hemorrhoid excised. ○ The wound is then closed with a running absorbable suture. ○ All three hemorrhoidal cushions may be removed using this technique; ■ however, care should be taken to avoid resecting a large area of perianal skin in order to avoid postoperative anal stenosis
  • 78. Treatment ● Open Hemorrhoidectomy ○ This technique, often called the Milligan and Morgan hemorrhoidectomy, follows the same principles of excision described earlier, but the wounds are left open and allowed to heal by secondary intention. ● Whitehead’s Hemorrhoidectomy ○ Whitehead’s hemorrhoidectomy involves circumferential excision of the hemorrhoidal cushions just proximal to the dentate line. ○ After excision, the rectal mucosa is then advanced and sutured to the dentate line ○ Risk of ectropion (Whitehead’s deformity).
  • 79. Treatment ● Procedure for Prolapse and Hemorrhoids/Stapled Hemorrhoidectomy ○ Procedure for prolapse and hemorrhoids (PPH) is also referred to as a stapled hemorrhoidopexy. ○ Best suited for patients with second and third-degree hemorrhoids, this out- patient procedure uses a stapling device similar in appearance and mechanism of action to an end-to-end anastomotic (EEA) stapling device used for rectal surgery. ○ Just as with an EEA stapler, proximal and distal tissue donuts, in this case consisting of mucosa and submucosa, are generated by the PPH stapler though the primary means by which this procedure provides relief for internal hemorrhoids is by pexying the redundant hemorrhoidal tissue, ligating the venules feeding the hemorrhoidal plexus and fixing redundant mucosa proximal to the dentate line. ○ Several studies suggest that this procedure is safe and effective, that it is associated with less postoperative pain and disability, and that it has an equivalent risk of postoperative complications when compared to excisional hemorrhoidectomy.
  • 80. Anal Fissure ● A fissure in ano is a tear in the anoderm distal to the dentate line. ● The pathophysiology of anal fissure is thought to be related to trauma from either the passage of hard stool or prolonged diarrhea. ● A tear in the anoderm causes spasm of the internal anal sphincter, which results in pain, increased tearing, and decreased blood supply to the anoderm. ○ This cycle of pain, spasm, and ischemia contributes to development of a poorly healing wound that becomes a chronic fissure. ○ The vast majority of anal fissures occur in the posterior midline. ○ Ten percent to 15% occur in the anterior midline. Less than 1% of fissures occur off midline
  • 81. Anal Fissure ● Anal fissure is extremely common. ● Characteristic symptoms include tearing pain with defecation and hematochezia (usually described as blood on the toilet paper). ● Patients may also complain of a sensation of intense and painful anal spasm lasting for several hours after a bowel movement. ● On physical examination, the fissure can often be seen in the anoderm by gently separating the buttocks. ● Patients are often too tender to tolerate digital rectal examination, anoscopy, or proctoscopy. ● An acute fissure is a superficial tear of the distal anoderm and almost always heals with medical management.
  • 82. Anal Fissure ● Chronic fissures develop ulceration and heaped-up edges with the white fibers of the internal anal sphincter visible at the base of the ulcer. ● There often is an associated external skin tag and/or a hypertrophied anal papilla internally. ● These fissures are more challenging to treat and may require surgery. ● A lateral location of a chronic anal fissure may be evidence of an underlying disease such as Crohn’s disease, HIV, syphilis, tuberculosis, or leukemia. ● If the diagnosis is in doubt or there is suspicion of another cause for the perianal pain such as abscess or fistula, an examination under anesthesia may be necessary.
  • 83. Treatment ○ Therapy focuses on breaking the cycle of pain, spasm, and ischemia thought to be responsible for development of fissure in ano. ○ First-line therapy to minimize anal trauma includes bulk agents, stool softeners, and warm sitz baths. ○ The addition of 2% lidocaine jelly or other analgesic creams can provide additional symptomatic relief. ○ Nitroglycerin ointment has been used locally to improve blood flow but often causes severe headaches. ○ Both oral and topical calcium channel blockers (diltiazem and nifedipine) have also been used to heal fissures and may have fewer side effects than topical nitrates. ○ Newer agents, such as arginine (a nitric oxide donor) and topical bethanechol (a muscarinic agonist), have also been used to treat fissures. ○ Medical therapy is effective in most acute fissures, but it will heal only approximately 50% of chronic fissures.
  • 84. ● Botulinum toxin (Botox) causes temporary muscle paralysis by preventing acetylcholine release from presynaptic nerve terminals. ● Injection of botulinum toxin is used in some centers as an alternative to surgical sphincterotomy for chronic fissure. ● Although there are few long-term complications from the use of botulinum toxin, healing appears to be equivalent to other medical therapies. ● Surgical therapy has traditionally been recommended for chronic fissures that have failed medical therapy, and lateral internal sphincterotomy is the procedure of choice. ● The aim of this procedure is to decrease spasm of the internal sphincter by dividing a portion of the muscle. ● Approximately 30% of the internal sphincter fibers are divided laterally by using either an open or closed technique. ● Healing is achieved in more than 95% of patients using this technique, and most patients experience immediate pain relief. ● Recurrence occurs in less than 10% of patients, and the risk of incontinence (usually to flatus) ranges from 5% to 15%. ● Advancement flaps (VY) with or without sphincterotomy have also been reported to successfully treat chronic fissures.
  • 85. Perirectal Abscess ● The majority of anorectal suppurative disease results from infections of the anal glands (cryptoglandular infection) found in the intersphincteric plane. ● Their ducts traverse the internal sphincter and empty into the anal crypts at the level of the dentate line. ● Infection of an anal gland results in the formation of an abscess that enlarges and spreads along one of several planes in the perianal and perirectal spaces. ● The perianal space surrounds the anus and laterally becomes continuous with the fat of the buttocks. ● The intersphincteric space separates the internal and external anal sphincters. ● It is continuous with the perianal space distally and extends cephalad into the rectal wall. ● The ischiorectal space (ischiorectal fossa) is located lateral and posterior to the anus and is bounded medially by the external sphincter, laterally by the ischium, superiorly by the levator ani, and inferiorly by the transverse septum.
  • 86. ● The ischiorectal space contains the inferior rectal vessels and lymphatics. ● The two ischiorectal spaces connect posteriorly above the anococcygeal ligament but below the levator ani muscle, forming the deep postanal space. ● The supralevator spaces lie above the levator ani on either side of the rectum and communicate posteriorly. ● The anatomy of these spaces influences the location and spread of cryptoglandular infection ● As an abscess enlarges, it spreads in one of several directions. ● A perianal abscess is the most common manifestation and appears as a painful swelling at the anal verge. Spread through the external sphincter below the level of the puborectalis produces an ischiorectal abscess. ● These abscesses may become extremely large and may not be visible in the perianal region.
  • 87. ● Digital rectal exam will reveal a painful swelling laterally in the ischiorectal fossa. Intersphincteric abscesses occur in the intersphincteric space and are notoriously difficult to diagnose, often requiring an examination under anesthesia. ● Pelvic and supralevator abscesses are uncommon and may result from extension of an intersphincteric or ischiorectal abscess upward or extension of an intraperitoneal abscess downward. ● Diagnosis ● Severe anal pain is the most common presenting complaint. ● A palpable mass is often detected by inspection of the perianal area or by digital rectal examination. Occasionally, patients will present with fever, urinary retention, or life-threatening sepsis.
  • 88. Diagnosis and Treatment ● The diagnosis of a perianal or ischiorectal abscess can usually be made with physical exam alone (either in the office or in the operating room). However, complex or atypical presentations may require imaging studies such as CT or MRI to fully delineate the anatomy of the abscess. ● Anorectal abscesses should be treated by drainage as soon as the diagnosis is established. ○ If the diagnosis is in question, an examination and drainage under anesthesia are often the most expeditious ways both to confirm the diagnosis and to treat the problem. ○ Delayed or inadequate treatment may occasionally cause extensive and life- threatening suppuration with massive tissue necrosis and septicemia. ○ Antibiotics are only indicated if there is extensive overlying cellulitis or if the patient is immunocompromised, has diabetes mellitus, or has valvular heart disease. ○ Antibiotics alone are ineffective at treating perianal or perirectal infection.
  • 89. Perianal abscess ● Most perianal abscesses can be drained under local anesthesia in the office, clinic, or emergency department. ● Larger, more complicated abscesses may require drainage in the operating room. ● A skin incision is created, and a disk of skin excised to prevent premature closure. ● No packing is necessary, and sitz baths are started the next day Intersphincteric abscess ● Intersphincteric abscesses are notoriously difficult to diagnose because they produce little swelling and few perianal signs of infection. ● Pain is typically described as being deep and “up inside” the anal area and is usually exacerbated by coughing or sneezing. ● The pain is so intense that it usually precludes a digital rectal examination. ● The diagnosis is made based on a high index of suspicion and usually requires an examination under anesthesia. ● Once identified, an intersphincteric abscess can be drained through a limited, usually posterior, internal sphincterotomy.
  • 90. Ischiorectal abscess ● An ischiorectal abscess causes diffuse swelling in the ischiorectal fossa that may involve one or both sides, forming a “horseshoe” abscess. ● Simple ischiorectal abscesses are drained through an incision in the overlying skin. ● Horseshoe abscesses require drainage of the deep postanal space and often require counter incisions over one or both ischiorectal spaces
  • 91. Supralevator Abscess ● This type of abscess is uncommon and can be difficult to diagnose. ○ Because of its proximity to the peritoneal cavity, supralevator abscesses can mimic intra-abdominal conditions. ● Digital rectal examination may reveal an indurated, bulging mass above the anorectal ring. ● It is essential to identify the origin of a supralevator abscess prior to treatment. ● If the abscess is secondary to an upward extension of an intersphincteric abscess, it should be drained through the rectum. ● If it is drained through the ischiorectal fossa, a complicated, suprasphincteric fistula may result. ● If a supralevator abscess arises from the upward extension of an ischiorectal abscess, it should be drained through the ischiorectal fossa. ● Drainage of this type of abscess through the rectum may result in an extrasphincteric fistula. ● If the abscess is secondary to intra-abdominal disease, the primary process requires treatment and the abscess is drained via the most direct route (transabdominally, rectally, or through the ischiorectal fossa).
  • 92. Proctitis ● Bacterial Infections ○ Proctitis is a common symptom of anorectal bacterial infection. ○ Neisseria gonorrhoeae is the most common bacterial cause of proctitis and causes pain, tenesmus, rectal bleeding, and mucus discharge. ○ Chlamydia trachomatis infection may be asymptomatic or may produce similar symptoms. ○ Treponema pallidum, the microbe causing syphilis, causes a chancre at the site of inoculation, which may be asymptomatic or may present as an atypical fissure (primary syphilis). ○ Condyloma lata are characteristic of secondary syphilis. ○ Chancroid, caused by Haemophilus ducreyi, is a disease manifested by multiple painful, bleeding lesions. ○ Inguinal lymphadenopathy and fluctuant, draining lymph nodes are characteristic. Donovania granulomatis infection produces shiny, red masses on the perineum (granuloma inguinale). ○ Diarrheal illnesses caused by organisms such as Campylobacter or Shigella may also be sexually transmitted. ○ Treatment consists of antimicrobial agents directed against the infecting organism.
  • 93. Proctitis ● Parasitic Infections ○ Entamoeba histolytica is an increasingly common sexually transmitted disease. ○ Amoebas produce ulcerations in the gastrointestinal mucosa and can infect any part of the gut. ○ Symptoms include diarrhea, abdominal pain, and tenesmus. ○ Giardia lamblia is also common and produces diarrhea, abdominal pain, and malaise. ● Viral Infections ○ Herpes proctitis is extremely common. ○ Proctitis is usually caused by type 2 herpes simplex virus and less commonly by type 1 herpes simplex virus. ○ Patients complain of severe, intractable perianal pain and tenesmus. ○ Pain often precedes the development of characteristic vesicles, and these patients may require an examination under anesthesia to exclude another diagnosis such as an intersphincteric abscess. ○ Diagnosis is confirmed by viral culture of tissue or vesicular fluid.
  • 94. Anal condyloma ● Condylomas occur in the perianal area or in the squamous epithelium of the anal canal. ● Occasionally, the mucosa of the lower rectum may be affected
  • 95. Anal condyloma ● HPV causes condyloma acuminata (anogenital warts) and is associated with squamous intraepithelial lesions and squamous cell carcinoma ● HPV types 6 and 11 commonly cause warts, but do not appear to cause malignant degeneration ● Treatment ○ Small warts on the perianal skin and distal anal canal ■ may be treated in the office with topical application of bichloracetic acid or podophyllin. ● Although 60% to 80% of patients will respond to these agents, recurrence and reinfection are common. ○ Imiquimod (Aldara): immunomodulator ■ highly effective in treating condyloma located on the perianal skin and distal anal canal. ○ Larger and/ or more numerous warts ■ excision and/or fulguration in the operating room. ■ Excised warts should be sent for pathologic examination to rule out dysplasia or malignancy. ○ The recent introduction of a vaccine against HPV holds promise for preventing anogenital condylomas
  • 96. Pilonidal disease ● it is speculated that the cleft creates a suction that draws hair into the midline pits when a patient sits. ● These ingrown hairs may then become infected and present acutely as an abscess in the sacrococcygeal region
  • 97. Pilonidal disease ● Once an acute episode has resolved, recurrence is common. ● An acute abscess should be incised and drained as soon as the diagnosis is made. ● Because these abscesses are usually very superficial, this procedure can often be performed in the office, clinic, or emergency department under local anesthetic. ● Because midline wounds in the region heal poorly, some surgeons recommend using an incision lateral to the intergluteal cleft.
  • 98. Pilonidal disease ● Treatment ○ The simplest method involves unroofing the tract, curetting the base, and marsupializing the wound. ○ The wound must then be kept clean and free of hair until healing is complete (often requiring weekly office visits for wound care). ○ Alternatively, a small lateral incision can be created and the pit excised. ■ effective for most primary pilonidal sinuses. ○ Complex and/or recurrent sinus tracts ■ require more extensive resection and closure with a Z-plasty, advancement flap, or rotational flap
  • 99. Fistula-in-ano ● The fistula usually originates in the infected crypt (internal opening) and tracks to the external opening, usually the site of prior drainage
  • 100. Fistula-in-ano Diagnosis ● Patients present with persistent drainage from the internal and/or external openings ● An indurated tract is often palpable. ● external opening is often easily identifiable ● identification of the internal opening may be more challenging. ● Goodsall’s rule can be used as a guide in determining the location of the internal opening
  • 101. Fistula-in-ano ● fistulas with an external opening anteriorly connect to the internal opening by a short, radial tract. ● Fistulas with an external opening posteriorly track in a curvilinear fashion to the posterior midline ● Categories: ○ intersphincteric fistula ■ tracks through the distal internal sphincter and intersphincteric space to an external opening near the anal verge ○ transsphincteric fistula ■ results from an ischiorectal abscess and extends through both the internal and external sphincters ○ suprasphincteric fistula ■ originates in the intersphincteric plane and tracks up and around the entire external sphincter ○ extrasphincteric fistula ■ originates in the rectal wall and tracks around both sphincters to exit laterally, usually in the ischiorectal fossa
  • 102. Fistula-in-ano Treatment ● surgical treatment is dictated by the location of the internal and external openings and the course of the fistula. ● Simple intersphincteric fistulas ○ can often be treated by fistulotomy (opening the fistulous tract), curettage, and healing by secondary intention ● transsphincteric fistula ○ Fistulas that include less than 30% of the sphincter muscles ■ treated by sphincterotomy ○ High transsphincteric fistulas, which encircle a greater amount of muscle ■ more safely treated by initial placement of a seton. ● suprasphincteric fistulas ○ treated with seton placement ● Extrasphincteric fistulas ○ treatment depends on both the anatomy of the fistula and its etiology
  • 103. Fistula-in-ano ● Complex fistulas with multiple tracts may require numerous procedures to control sepsis and facilitate healing. ● Failure to heal may ultimately require fecal diversion ● Proctoscopy ○ performed in all cases of complex and/or nonhealing fistulas to assess the health of the rectal mucosa. ● Biopsies of the fistula tract should be taken to rule out malignancy. ● Higher fistulas may be treated by an endorectal advancement flap
  • 104. Anorectal CA ● Neoplasms of the anal canal have traditionally been divided into those affecting the anal margin (distal to the dentate line) and those affecting the anal canal (proximal to the dentate line) based on lymphatic drainage patterns. ● Lymphatics from the anal canal proximal to the dentate line drain cephalad via the superior rectal lymphatics to the inferior mesenteric nodes and laterally along both the middle rectal vessels and inferior rectal vessels through the ischiorectal fossa to the internal iliac nodes. ● Lymph from the anal canal distal to the dentate line usually drains to the inguinal nodes. ● It can also drain to the superior rectal lymph nodes or along the inferior rectal lymphatics to the ischiorectal fossa if primary drainage routes are blocked with tumor
  • 105. Anorectal CA Squamous intraepithelial lesions ● High-grade lesions are precursors to invasive squamous cell carcinoma (epidermoid carcinoma) and may appear as a plaque or may only be apparent with high-resolution anoscopy and application of acetic acid and/or Lugol’s iodine solution. ● The incidence of both squamous intraepithelial lesions and epidermoid carcinoma of the anus has increased dramatically among human immunodeficiency virus (HIV)– positive men who have sex with men. ● This increase is thought to result from increased rates of HPV infection along with HIV-induced immunosuppression
  • 106. Anorectal CA ● Treatment ○ ablation ■ high-grade dysplasia ■ Because of a high recurrence and/or reinfection rate, these patients require close surveillance. ○ High-risk patients should be followed with frequent anal Papanicolaou (Pap) smears every 3 to 6 months. ■ abnormal Pap smear should be followed by an examination under anesthesia and anal mapping using highresolution anoscopy. ○ High-resolution anoscopy shows areas with abnormal telangiectasias that are consistent with high grade dysplasia. ○ Medical therapy for HPV has also been proposed. ■ Topical immunomodulators such as imiquimod (Aldara) have been shown to induce regression in some series. ■ Topical 5-fluorouracil has also been used in this setting. ○ Finally, the introduction of a vaccine against HPV may help decrease the incidence of this disease in the future
  • 107. Anorectal CA Epidermoid Carcinoma. ● slow-growing tumor and usually presents as an intra-anal or perianal mass. ● Pain and bleeding may be present. ● Treatment ○ Perianal epidermoid carcinoma may be treated in a similar fashion as squamous cell carcinoma of the skin in other locations because wide local excision can usually be achieved without resecting the anal sphincter. ○ Intra-anal epidermoid carcinoma cannot be excised locally ■ first-line therapy relies on chemotherapy and radiation (the Nigro protocol: 5- fluorouracil, mitomycin C, and 30 Gy of external beam radiation). ○ It is important to note that epidermoid carcinomas continue to respond after completion of chemoradiation. ○ Lesions that persist greater than 3 to 6 months after therapy may represent persistent disease and should be biopsied. ○ Recurrence: requires radical resection (APR). ○ Metastasis to inguinal lymph nodes is a poor prognostic sign
  • 108. Anorectal CA Verrucous Carcinoma (Buschke-Lowenstein Tumor, Giant Condyloma Acuminata). ● locally aggressive form of condyloma acuminata. ● Although these lesions do not metastasize, they can cause extensive local tissue destruction ● may be grossly indistinguishable from epidermoid carcinoma. ● Treatment ○ Wide local excision (treatment of choice) ○ radical resection ○ Topical immunomodulators such as imiquimod (Aldara) ○ may shrink some tumors, but they are almost never curative. ○ Very large lesions: external beam radiation, but resection is almost always required.
  • 109. Anorectal CA Basal Cell Carcinoma ● rare and resembles basal cell carcinoma elsewhere on the skin (raised, pearly edges with central ulceration). ● slowgrowing tumor that rarely metastasizes. ● Treatment ○ Wide local excision (treatment of choice) ■ recurrence occurs in up to 30% of patients. ○ Radical resection and/or radiation therapy ■ for large lesions
  • 110. Anorectal CA Adenocarcinoma ● extremely rare and usually represents downward spread of a low rectal adenocarcinoma ● may occasionally arise from the anal glands or may develop in a chronic fistula. ● Treatment ○ Radical resection, usually after neoadjuvant chemoradiation ● Extramammary perianal Paget’s disease ○ is adenocarcinoma in situ arising from the apocrine glands of the perianal area ○ typically plaque-like and may be indistinguishable from high-grade intraepithelial lesions. ○ Histology: Paget’s cells are seen. ○ Treatment ■ These tumors are often associated with a synchronous gastrointestinal adenocarcinoma, so a complete evaluation of the intestinal tract should be performed. ■ Wide local excision ● for perianal Paget’s disease
  • 111. Anorectal CA Melanoma ● Anorectal melanoma is rare ○ < 1% of all anorectal malignancies ○ 1% to 2% of melanomas. ● Treatment ○ prognosis for patients with anorectal disease: poor. ■ Overall 5-year survival: <10% ○ A few patients with anorectal melanoma ■ present with isolated local or locoregional disease that is potentially resectable for cure, and both radical resection (APR) and wide local excision have been advocated. ○ Recurrence is common ○ wide local excision: for initial treatment of localized anal melanoma ○ The addition of adjuvant chemotherapy, biochemotherapy, vaccines, or radiotherapy may be of benefit in some patients, but efficacy remains unproven
  • 112. Case 1: A 33 year-old female comes to see you complaining of perirectal pain
  • 114.
  • 115. B) What pertinent questions will you ask during history taking? Why? • Pain  How long has it been present?  Is it constant?  What makes it better or worse?  Is the pain increasing or decreasing?  What is the quality of the pain?  Patients with thrombosed or incarcerated (non-reducible prolapsed) hemorrhoids usually present with severe, constant pain that has come on suddenly. Often, patients will recall an episode of severe constipation or lifting heavy objects that preceded the pain.
  • 116.  Another diagnosis that has a characteristic pain quality is an anal fissure. These symptoms also often present after an episode of severe constipation or anal trauma, and are characterized as a very sharp, cutting or tearing pain, often described as “passing glass” during defecation or having “a sharp knife poking” the anus. They may also complain of burning for hours after. Often, patients with a fissure note that prior to their bowel movement, they did not have pain.  Pain that is constant but comes on gradually over the course of several days is characteristic of a perianal or perirectal abscess or an anal sexually transmitted disease (i.e., syphilis or herpes).  Pain that worsens over many weeks or months is typical of proctitis and malignancies.  In general, moderate or mild hemorrhoidal disease is not associated with significant pain, though patients may report some discomfort or itching in the area.
  • 117. • Presence of mass?  Is there a mass or swelling noted by the patient?  Is it new?  Is it enlarging?  Is it always present or does it at times disappear?  Is there more than one mass?  Patients with the most common types of perirectal abscess, pilonidal abscess, and thrombosed external or incarcerated hemorrhoids will appreciate a new mass or swelling which is tender to touch.  Anal fissures can be associated with an anal skin tag (also known as a sentinel pile) that patients may notice.
  • 118.  Patients with intermittent grade II or grade III hemorrhoids can have protrusion of tissue.  Patients with anal condyloma can also note new masses, which tend to be small and multiple.  Other more concerning things can also present as a new mass, including anal cancers  Less commonly, rectal prolapse can also present as a new large mass that can be confused with hemorrhoids
  • 119. • Presence of bleeding?  How much?  What is the location of the blood: on the toilet paper? In the toilet water?  On top of the stool, or mixed in with the stool?  Are there symptoms of anemia?  Internal hemorrhoids classically bleed with bowel movements, resulting in blood on the tissue or in the toilet water and coating the stools. Sometimes, the bleeding can be severe enough to cause anemia, though generally, it is mild.  Anal fissures also have a similar bleeding pattern, though these are often associated with pain.
  • 120. • Proctitis patients classically will have urgency and frequency and often have frequent, small, bloody bowel movements. The bleeding can be bright or darker red. Bleeding can occur with pilonidal disease if there is a break in the skin, though the bleeding is mild and usually the patient can appreciate the bleeding is not near the anus but at the top of the gluteal crease, not related to bowel movements and frequently just located on the underwear.  Thrombosed external hemorrhoids may have mild bleeding seen on the toilet paper or in the underwear.  Malignancies often bleed with even gentle touch or manipulation.  Occasionally, if a perianal abscess has developed into a perianal fistula, the external opening of the fistula can also have mild bleeding.
  • 121. • Presence of drainage?  How much?  What is the character?  The classic draining lesion in the perianal region is a perianal fistula, which produces scant, thick yellow or greenish-tinged discharge.  Abscesses that have spontaneously opened can produce some drainage, which is usually copious at first and rapidly decreases in volume.  Prolapsed internal hemorrhoids or rectal prolapse can also produce some drainage, though this tends to be thin, white or clear drainage and occasionally pink-tinged
  • 122. • Other questions?  There are other important questions to ask for patients with any perianal problem, including inquiring about abdominal pain and any change in their bowel habits, including what they consider normal and regular.  Additionally, soft but formed stool is the ideal, which should require little to no straining to evacuate. • It is also important to inquire about the patients’ control of their bowels and any accidents and leakage they may have, as fecal incontinence is a frequent problem in older women due to previous obstetrical injuries during vaginal deliveries
  • 123. C) How do you do digital rectal exam? •Introduction •Introduce yourself with your name, role and confirm the patient’s name and date of birth. Briefly explain the procedure and the degree of exposure required. The patient should be undressed from the waist down including any undergarments. •Obtain consent and proceed to wash hands. For this examination, it is essential to have a chaperone. •Position the bed flat and then put on a pair of non-sterile gloves and an apron. Instruct the patient to lie on the bed in the left lateral position with their knees bent up to their chest. Offer the patient a blanket to maintain their dignity. Ask if they are in any pain and are comfortable.
  • 124. •Equipment •Before you begin your examination, prepare the following equipment in a plastic tray:  Non-sterile gloves.  Disposable apron.  Tissues.  Lubricating jelly.
  • 125. •Inspection •Warn the patient that you are about to look around the area. •Separate the buttocks using your non-dominant hand and inspect the anus and natal cleft. Inspect for the following signs:  External haemorrhoids  Skin tags (fleshy extra skin hanging from anus): These are usually harmless growths that may be mistaken for warts or haemorrhoids. They can be caused by inflammation, injury or leftover skin after haemorrhoid removal.  Excoriation marks (scratch marks): These are as a result of anal itchiness, which can be due infection, haemorrhoids, bowel incontinence, long-term constipation/diarrhoea or repeated glyceryl trinitrate cream topical application (used to treat anal fissures).  Anal fissures (skin tear): These occur with long term straining of hard faeces such as with chronic constipation or in inflammatory bowel disease.  Anal fistulae (small opening often with discharge): These can also be present in inflammatory bowel disease (particularly Crohn’s disease). Other causes include abscesses that have failed to heal and diverticulitis.  Ask the patient to cough and check for any internal haemorrhoids or rectal prolapse.
  • 126. •Anal tone •Lubricate the index finger of your dominant hand with some lubricating jelly and place your other hand on the patient’s hip. Place one finger on the anal orifice and lightly palpate for induration (a hardened/fibrous section under the skin), which is associated with inflammatory conditions such as Crohn’s disease. •Ask the patient to breathe in deeply and then relax to loosen the sphincter. •Warn the patient that they may feel some cold jelly, and then advance your index finger into the anal canal. Ask the patient to bear down on your finger to assess anal tone. Decreased anal tone may be due to long-term diarrhoea, diabetes, spinal trauma or simply old age.
  • 127. Palpation •Anal canal •Sweep 360° around the full rectum. The anterior walls may be more easily examined by rotating your body and wrist. Check for any lumps; these may be haemorrhoids, polyps or a tumour. Feel for palpable faeces and tenderness. If the patient is in severe pain during the examination, consider an anal fissure, abscess or ulcer. •Note the location, using a clock face, and texture of anything you find. e.g. 1cm irregular mass at 9 o’clock. •After advancing your finger, it may be worth pausing for a few seconds to allow the patient to acclimatise and relax. •During palpation look at the patient’s face for signs of distress/pain.
  • 128. •Prostate •With males, identify the prostate, its central sulcus and assess the size. A normal prostate is smooth and walnut sized. Check for symmetry, the presence of any nodules and any tenderness. Abnormalities in these features can give indications towards pathology:  Enlarged with a deep sulcus: Benign prostatic hyperplasia.  Enlarged and very tender: Prostatitis.  Enlarged with a rough nodule: Unilateral cancer.  Hard, asymmetrical and irregular with an impalpable sulcus: Prostate cancer.
  • 129. •Completion •Inspection of the gloved finger •Withdraw and inspect the gloved finger for blood or mucus, suggesting ulcerative colitis. Thank the patient, remove your gloves and place them in a clinical waste bin. Wash your hands. •Wipe away the jelly from the anus and offer the patient some extra tissues for their own comfort. Give the patient privacy to get dressed. •Post-examination •Offer to take a focused history, an abdominal exam and the appropriate tests depending on the pathology.
  • 130. D) What will you request for further diagnostics? And why? 1) Anoscopic examination: Most specific and conclusive diagnostic test for hemorrhoids  An anoscope is approximately 7cm long. It comes in various sizes and shapes. This does not require bowel prep, nor does it require sedation, as this can be easily done in the office for most patients.  If a patient has significant pain on exam, and a cause for the pain cannot be determined in the office, then an exam with sedation can be done in the GI lab or in the operating room, if needed.  The anoscope allows one to see the whole anal canal and, depending on patient habitus and type of anoscope used, it can also allow one to see the distal rectum for 2- 4cm above the dentate line.  It is helpful to evaluate internal hemorrhoids, the extent of a small anal cancer or anal condyloma within the anal canal, as well as to look for internal fistula openings.
  • 131. 2) Colonoscopy/flexible sigmoidoscopy: Usually normal in hemorrhoids; may reveal other pathologies  Used to exclude serious pathology such as inflammatory bowel disease or cancer  In presence of suspicious symptoms such as altered bowel habit (diarrhea and/or constipation), abdominal pain, weight loss, iron deficiency anemia or passage of blood clots and/or mucus, lower gastrointestinal endoscopy is performed.  Definitive test is colonoscopy • If flexible sigmoidoscopy is chosen, it should be combined with barium enema to assess proximal colon in high risk patients(eg: family history of bowel cancer)
  • 132. 3) PROCTOSCOPY: A proctoscope may also be referred to as a rigid sigmoidoscope or rigid proctoscope and is 25cm long. As the average rectum is 15cm long, it is obvious that with full insertion,  the lower part of the sigmoid can be seen.  This does not require a bowel prep, but for best evaluation, a patient will perform one or two enemas prior to the procedure to allow the rectum to be free from stool. This also does not require sedation if the patient does not have severe pain or anxiety, and can be routinely done in the office.  Patients may be positioned in the knee-chest position but more frequently, they are either positioned in a lateral decubitus position or on a procto table.  This is frequently performed to evaluate malignancies that may be extending more proximal than what can be seen by an anoscope.  This is the standard technique used to measure the distal edge of a higher tumor from the anal verge, as is done for rectal cancers, to determine the location in the rectum.  The rigid scope allows a straight measurement to be taken, unlike a flexible scope that can lead to inaccuracies due to looping or flexing of the scope. Since the entire rectum can be visualized, this is an ideal scope to evaluate for proctitis and to perform biopsies of any lesions in the rectum
  • 133. • 4) FBC:Ordered only if there is concern that the patient has experienced significant prolonged rectal bleeding and signs of anemia are present • 5) Stool for occult blood: Unnecessary unless no significant hemorrhoidal tissue is seen on examination; Further evaluation deemed unnecessary if the results are negative
  • 134. E) How will you manage this patient? (Based on differential diagnosis) • HEMORRHOIDS TREATMENT  Medical Therapy  improves with the addition of dietary fiber, stool softeners, increased fluid intake, and avoidance of straining.  pruritus often may improve with improved hygiene.  Rubber Band Ligation .  first-, second-, and selected third-degree hemorrhoids may be treated by rubber band ligation Mucosa located 1 to 2 cm proximal to the dentate line is grasped and pulled into a rubber band applier. After firing the ligator, the rubber band strangulates the underlying tissue, causing scarring and preventing further bleeding or prolapse
  • 135.  Infrared Photocoagulation  treatment for small first- and second-degree hemorrhoids.  The instrument is applied to the apex of each hemorrhoid to coagulate the underlying plexus.Larger hemorrhoids and with a significant amount of prolapse are not effectively treated with this technique.  Sclerotherapy The injection of bleeding internal hemorrhoids with sclerosing agents is another effective office technique for treatment of first-, second-, and some third-degree hemorrhoids.  One to 3 mL of a sclerosing solution (phenol in olive oil, sodium morrhuate, or quinine urea) is injected into the submucosa of each hemorrhoid.  Excision of Thrombosed External Hemorrhoids Acutely thrombosed external hemorrhoids- cause intense pain and a palpable perianal mass during the first 24 to 72 hours after thrombosis.  The thrombosis - elliptical excision performed in the office under local anesthesia. Because the clot is usually loculated, simple incision and drainage is rarely effective  . After 72 hours, the clot begins to resorb, and the pain resolves spontaneously.  Excision is unnecessary, but sitz baths and analgesics are often helpful.
  • 136.  Operative Hemorrhoidectomy  Closed Submucosal Hemorrhoidectomy  the Parks or Ferguson hemorrhoidectomy involves resection of hemorrhoidal tissue and closure of the wounds with absorbable suture.  Open Hemorrhoidectomy This technique, often called the Milligan and Morgan hemorrhoidectomy, follows the same principles of excision described earlier, but the wounds are left open and allowed to heal by secondary intention. • Whitehead’s Hemorrhoidectomy involves circumferential excision of the hemorrhoidal cushions just proximal to the dentate line. After excision, the rectal mucosa is then advanced and sutured to the dentate line. risk of ectropion (Whitehead’s deformity).
  • 137.  Anal fissure: First-line therapy -bulk agents, stool softeners, and warm sitz baths. The 2% lidocaine jelly or other analgesic creams can provide symptomatic relief.  Nitroglycerin ointment -improve blood flow  Both oral and topical calcium channel blockers (diltiazem and nifedipine) have also been used to heal fissures  Botulinum toxin (Botox) causes temporary muscle paralysis. It is an alternative to surgical sphincterotomy for chronic fissure.  Surgical therapy -lateral internal sphincterotomy The aim of this procedure is to decrease spasm of the internal sphincter -open or closed technique.
  • 138.  Rectovaginal fistula: The treatment of rectovaginal fistula depends on the size, location, etiology, and condition of surrounding tissues.  Because up to 50% of fistulas caused by obstetric injury heal spontaneously  Low and mid-rectovaginal fistulas are usually best treated with an endorectal advancement flap  Fistulas caused by malignancy should be treated with resection of the tumor  Perianal abscess: Most perianal abscesses can be Treated By draining under local anesthesia in the office, clinic, or emergency department.  Larger, more complicated abscesses may require drainage in the operating room  A skin incision is created, and a disk of skin excised to prevent premature closure
  • 139. Case 2: A 63 year old woman presents to the office for evaluation of a painful anal mass
  • 140. A) What pertinent questions will you ask during history taking? Why? • HISTORY OF PRESENT ILLNESS  Onset of pain o Acute-ischemia ,meckels diverticulum,angiodysplasia o Chronic—IBD,polyp  Location of mass o If near tail bone-pilondial sinus o In or around anal opening-anal condhyloma
  • 141.  Characteristic of pain o Tearing pain- anal fissure o Severe pain-abcess,hemorrhoid(external),fistula o Painless-polyp, meckles diverticulum o Pain on passing stools - haemorrhoid  Characteristic of mass o Hard mass –ischemic proctitis o Multi numbered, increased in size,irregular mass-anal condyloma o Hair growth-pilondial disease
  • 142.  Associated symptoms o Fever , urinary retention-abcess o Bleeding- anal fissure,haemorrhoid,anal cancer,anal condyloma o Itching – haemorrhoid,anal cancer,anal condyloma o Spasm(lasting several hour after bowel movement)-anal fissure o Presene of skin tag-anal fissure anal condyloma o Abdominal pain & bloody diarrhea- IBD,infectious diarrhea, CA colon  Characteristic of stool o Hard stool-cause of haemorrhoid o Thin stool-anal cancer o Brown stool streaked with blood-anal cause  Any underlying disease o Infection-abcess o HPV- most common cause of anal condylom&anorectal cancer o Corhns disease,HIV,TB,syphilis,leukemia-chronic anal fissure
  • 143. • Past medical history  History of trauma to anal canal  Previous history of anorectal abcess o May develop fistula in ano  Any history of previous history, malignancy or STD o Anal cancer  Previous surgery in distal abdomen region o May cause anorectal abcess formation  History of trauma,radition,infection(TB,actinomycosis,Chlamydia) o May cause anorectal abcess formation  History of cardio vascular disease o Ischemic proctitis(rare) o Mesenteric embolism  Liver disease o Variceal bleeding
  • 144. • Family history  History of malignancy o Colorectal cancer  Adenomatous polyposis • Social history  Smoking o Risk factor for anal cancer  Alcohol o Increases the risk of colorectal cancer
  • 145. B) What further diagnostics will you request? • DRE: Severe anal pain is the most common presenting complaint. A palpable mass is often detected by inspection of the perianal area or by digital rectal examination. Physical examination may reveal necrotic skin, bullae,or crepitus. Digital rectal examination may reveal an indurated, bulging mass above the anorectal ring. • Anascopy : This procedure is used to examine the walls, veins and tissue of the anus. It also helpful to examine the surrounding structures of anus and part of the rectum.. • Proctoscopy : This procedure is used examine the distal rectum and anal canal and it is also helpful to do biopsy of the mass. • Mostly we can confirm our diagnosis by these procedures. • But Patients are often too tender to tolerate digital rectal examination, anoscopy, or proctoscopy. At this situation we have to go with Barium enema X-ray.
  • 146. C) How will you manage this patient? • Anorectal pain is most often secondary to  an anal fissure,  perirectal abscess and/or fistula  or a thrombosed hemorrhoid.  Other, less common causes of anorectal pain include anal canal neoplasms, perianal skin infection, and dermatologic conditions.  Proctalgia fugax results from levator spasm and may present without any other anorectal findings.
  • 147. • Anal fissure: First-line therapy to minimize anal trauma includes bulk agents, stool softeners, and warm sitz baths.  Nitroglycerin ointment has been used locally to improve blood flow but often causes severe headaches.  oral and topical calcium channel blockers (diltiazem and nifedipine) have also been used to heal fissures and may have fewer side effects than topical nitrates  Botulinum toxin (Botox) causes temporary muscle paralysis by preventing acetylcholine release from presynaptic nerve terminals.  Injection of botulinum toxin is used in some centers as an alternative to surgical sphincterotomy for chronic fissure. • Surgical therapy has traditionally been recommended for chronic fissures that have failed medical therapy
  • 148.  Anorectal abscesses: Should be treated by drainage as soon as the diagnosis is established.  If the diagnosis is in question, an examination and drainage under anesthesia are often the most expeditious ways both to confirm the diagnosis and to treat the problem  Antibiotics are only indicated if there is extensive overlying cellulitis or if the patient is immunocompromised, has diabetes mellitus, or has valvular heart disease.  surgical treatment is dictated by the location of the internal and external openings and the course of the fistula.  Simple intersphincteric fistulas can often be treated by fistulotomy (opening the fistulous tract), curettage, and healing by secondary intention
  • 149. • Fistula In Ano  surgical treatment is dictated by the location of the internal and external openings and the course of the fistula.  Simple intersphincteric fistulas can often be treated by fistulotomy (opening the fistulous tract), curettage, and healing by secondary intention.